Monday, March 17, 2008

Usmle-mcq's-9

)<1>A 68-year-old female who recently had a cholecystectomy develops a
fever of 103°F and has persistent drainage from her biliary catheter.
She is given cephalothin and gentamicin for 10 days. Her serum creatinine
level increases to 7.6 mg/dL. Her urine output is 1.3 L/day and has not
diminished over the past few days. There is no history of hypotension
and her vital signs are normal. Renal ultrasonography shows no evidence
of obstruction. The most likely etiology of the patient's condition is
A. acute glomerulonephritis
B. acute renal failure secondary to cephalothin
C. gentamicin nephrotoxicity
D. renal artery occlusion
E. sepsis
Answer
--------------------------------------------------------------------------------
<2>A 45-year-old man presents to a physician with back pain and facial pain.
Physical examination demonstrates coarse facial features and kyphosis.
Laboratory examination is remarkable for elevated alkaline phosphatase
. X-ray studies demonstrate skull thickening with narrowing of foramina,
and bowing of the femur and tibia. Bone biopsy reveals a mosaic
pattern of bone spicules with prominent osteoid seams. Which of
the following neoplasms occurs at an increased frequency in patients
with this disorder?
A. Astrocytoma
B. Hodgkin's lymphoma
C. Meningioma
D. Non-Hodgkin's lymphoma
E. Osteosarcoma
Answer
--------------------------------------------------------------------------------
<3> Which of the labeled structures is responsible for the secretion
of renin?
A. A
B. B
C. C
D. D
E. E
Answer

--------------------------------------------------------------------------------

<4>A 42-year-old African-American man sustains severe injuries in an
automobile accident and is admitted to the intensive care unit.
Examination of a peripheral blood smear on the 3rd day of admission
reveals helmet cells, schistocytes, and decreased platelets. Which of
the following is most strongly suggested by these findings?
A. Autoimmune hemolysis
B. Disseminated intravascular coagulation (DIC)
C. Hereditary spherocytosis
D. Megaloblastic anemia
E. Sickle cell anemia
Answer
--------------------------------------------------------------------------------
<5>A 12-month-old child is diagnosed with an atrial septal defect.
What is the most common cause of such a congenital heart malformation?
A. Failure of formation of the septum primum
B. Failure of formation of the septum secundum
C. Incomplete adhesion between the septum primum and septum secundum
D. Malformation of the membranous interventricular septum
E. Malformation of the muscular interventricular septum
Answer
--------------------------------------------------------------------------------
<6>A 72-year-old man with prostate cancer is treated with leuprolide.
What is the mechanism of action of this drug?
A. It inhibits 5a-reductase
B. It is a competitive antagonist at androgen receptors
C. It is a competitive inhibitor of LH
D. It is a synthetic analog of GnRH
E. It is a testosterone agonist
Answer
--------------------------------------------------------------------------------

<7>An XX genotypic infant is born with ambiguous genitalia. Laboratory
examination reveals hypoglycemia, hyperkalemia, and salt wasting. S
erum 17-OH progesterone is markedly increased. Which of the following is
the most likely diagnosis?
A. 5-alpha-reductase deficiency
B. 11-beta-hydroxylase deficiency
C. 17-alpha-hydroxylase deficiency
D. 21-hydroxylase deficiency
E. Complete androgen resistance
Answer
--------------------------------------------------------------------------------

<8>A 57-year-old fisherman with a history of alcoholism is hospitalized in
Gulfport, Mississippi with a 1-day history of severe, watery diarrhea after
eating several raw oysters. He is badly dehydrated on admission, and within
12 hours, he becomes severely hypotensive and dies. Which of the following
pathogens is the most likely cause of this man's death?
A. Citrobacter diversus
B. Enterotoxigenic E. coli
C. Providencia stuartii
D. Vibrio cholerae
E. Vibrio vulnificus
Answer
--------------------------------------------------------------------------------

<9>An Hispanic male is referred to the dermatology clinic of a major medical
center. On physical examination, the man has several disfiguring lesions on
his face and there is loss of cutaneous sensation to fine touch, pain, and
temperature. An acid-fast organism is observed in scrapings from a skin
lesion. Which of the following organisms is the most likely cause of this
patient's disease?
A. Bartonella henselae
B. Listeria monocytogenes
C. Mycobacterium avium-intracellulare
D. Mycobacterium leprae
E. Nocardia asteroides
Answer
--------------------------------------------------------------------------------

<10>Microscopic examination of a section of a normal young adult ovary
reveals large numbers of unusually large cells surrounded by a single
layer of flat epithelial cells. In which phase of the cell cycle are
these cells arrested?
A. Diplotene stage of the 1st meiotic division
B. Diplotene stage of the 2nd meiotic division
C. Metaphase stage of mitosis
D. Prophase stage of mitosis
E. Prophase stage of the 2nd meiotic division
Answer
--------------------------------------------------------------------------------

<11>A 3-year-old child is seen by a pediatrician because he has developed
multiple isolated lesions on his face and neck. Physical examination
reveals many lesions up to 4 cm in diameter with golden crusts, while
in other sites small blisters and weeping areas are seen. Which of the
following is the most likely diagnosis?
A. Aphthous ulcers
B. Erysipelas
C. Herpes simplex I
D. Impetigo
E. Measles
Answer
--------------------------------------------------------------------------------

<12>A 2-year-old-boy is brought to the pediatrician by his mother because
he has had several episodes of rectal bleeding. Evaluation with a
technetium-99m perfusion scan reveals a 3-cm ileal outpouching located
60 cm from the ileocecal valve. This structure likely contains which of
the following types of ectopic tissue?
A. Duodenal
B. Esophageal
C. Gastric
D. Hepatic
E. Jejunal
Answer
--------------------------------------------------------------------------------

<13>A 19-year-old college student presents to the student health clinic
complaining of weakness, malaise, and a chronic cough. He has a fever of
100 degrees F and a dry cough; no sputum can be obtained for laboratory
analysis, so a bronchial lavage is performed and the washings are
submitted to the laboratory. The laboratory reports that the organism
is "slow-growing." Serodiagnosis reveals Strep MG agglutinins in the
patient's serum. Which of the following organisms is the most likely
cause of this student's illness?
A. Klebsiella pneumoniae
B. Mycoplasma pneumoniae
C. Parainfluenza virus
D. Respiratory syncytial virus
E. Streptococcus pneumoniae
Answer
--------------------------------------------------------------------------------

<14>A 52-year-old male is brought into the emergency room by his wife
because he has been complaining of a severe headache. Physical exam
reveals ptosis of the right eyelid with the right eye facing down and out.
There is a fixed and dilated right pupil with an inability to accommodate.
Subarachnoid blood appears on noncontrast CT scan. Magnetic resonance
angiography (MRA) would be expected to reveal an aneurysm of which of
the following vessels?
A. Anterior cerebral artery
B. Anterior choroidal artery
C. Anterior communicating artery
D. Middle cerebral artery
E. Ophthalmic artery
F. Posterior communicating artery
G. Posterior inferior cerebellar artery
Answer
--------------------------------------------------------------------------------

<15>A 24-year-old graduate student presents to a physician with complaints
of severe muscle cramps and weakness with even mild exercise. Muscle
biopsy demonstrates glycogen accumulation, but hepatic biopsy is
unremarkable. Which of the following is the most likely diagnosis?
A. Hartnup's disease
B. Krabbe's disease
C. McArdle's disease
D. Niemann-Pick disease
E. Von Gierke's disease
Answer

--------------------------------------------------------------------------------

<16>A baby is born with a flat facial profile, prominent epicanthal folds,
and simian crease. She vomits when fed, and upper GI studies demonstrate
a "double bubble" in the upper abdomen. Which of the following
cardiovascular abnormalities might this child also have?
A. Atrial septal defect
B. Berry aneurysm
C. Coarctation of the aorta
D. Endocardial cushion defect
E. Tetralogy of Fallot
Answer
--------------------------------------------------------------------------------

<17>An asymptomatic, 24-year-old African-American woman in her second
trimester of pregnancy has the following laboratory findings: Based
on the laboratory data, which of the following tests is necessary for fur
ther evaluation of this patient?
A. Creatinine clearance
B. Oral glucose tolerance test
C. Serum ferritin
D. Sickle cell preparation
E. No further study is necessary
Answer
--------------------------------------------------------------------------------

<18>Endometrial biopsy demonstrates a thick endometrium with long,
coiled glands lined by a columnar epithelium with prominent cytoplasmic
vacuoles adjacent to the gland lumen. Earlier in the menstrual cycle,
the glands were much smaller and were lined with cells that did not
have vacuoles. Which of the following hormones is primarily responsible
for inducing this change in appearance?
A. Aldosterone
B. Cortisol
C. Estrogen
D. Progesterone
E. Thyroxine

Answer
--------------------------------------------------------------------------------

<19>A 28-year-old man discovers a mass in his neck while buttoning
his shirt collar. Physical examination reveals a 2 cm mass in one
thyroid lobe, which is "cold" on scintiscan. Aspiration of the
nodule demonstrates small "solid balls" of neoplastic follicular
cells. Careful examination of these tissue balls reveals that
they contain microscopic blood vessels and fibrous stroma in
their centers. Which of the following is the most likely diagnosis?
A. Follicular carcinoma
B. Hashimoto's disease
C. Medullary carcinoma
D. Papillary carcinoma
E. Thyroid adenoma
Answer
--------------------------------------------------------------------------------

<20>Nissl bodies correspond to which of the following cytoplasmic organelles?
A. Golgi apparatus
B. Mitochondria
C. Nucleoli
D. Rough endoplasmic reticulum
E. Smooth endoplasmic reticulum
Answer
--------------------------------------------------------------------------------

<21>A 40-year-old man with sleep apnea participates in a sleep study.
During his evaluation, normal sawtooth waves are observed on his EEG
tracing. This pattern is associated with which period of sleep?
A. REM
B. Stage 1
C. Stage 2
D. Stage 3
E. Stage 4
Answer
--------------------------------------------------------------------------------

<22>A United Nations representative from a poor African country has an
influenza-like illness that resolves in a few days. Less than a week later,
however, he develops muscle pain, spasms, and sensory disturbances.
Two days after this, flaccid paralysis occurs. Which of the following
is most likely to be immediately life-threatening in this patient?
A. Acute renal failure
B. Bowel paralysis
C. Fulminant liver failure
D. Gastrointestinal bleeding
E. Respiratory paralysis
Answer
--------------------------------------------------------------------------------

<23>What is the normal myeloid to erythroid ratio in bone marrow?
A. 1:1
B. 1:3
C. 1:10
D. 3:1
E. 10:1
Answer
--------------------------------------------------------------------------------

<24>A 45-year-old male complains of fatigue and increased frequency of
urination. Questioning reveals that he is somewhat confused. Physical
examination reveals a 5'10", 240 lb. individual whose fat is centrally
distributed. Urine is positive for glucose, but negative for ketones.
A blood sample drawn from this patient is likely to exhibit which of
the following compared to that of a normal individual?
A. Decreased concentration of C-peptide
B. Decreased pH
C. Increased antibodies against islet cell proteins
D. Increased concentration of b-hydroxybutyrate
E. Increased osmolarity
Answer
--------------------------------------------------------------------------------

<25>At which of the following ages does fetal movement first occur?
A. 1 month
B. 2 months
C. 4 months
D. 6 months
E. 7 months
Answer
--------------------------------------------------------------------------------

<26>A child with retinoblastoma is found to have a 13q14 deletion.
The Rb gene, which resides at this locus, produces which kind of
tumor-associated protein?
A. Cell cycle regulator
B. Growth factor
C. Growth factor-binding protein
D. Growth factor receptor
E. Transcription activator
Answer
--------------------------------------------------------------------------------

<27>A 29-year-old woman is involved in an automobile accident, and
is taken to the emergency room by paramedics. X-rays reveal a
fracture of her pelvis. During the healing of the pelvic fracture,
the obturator nerve becomes entrapped in the bone callus. Compression
of this nerve by the growing bone may result in weakness of which of
the following muscles?
A. Adductor magnus
B. Biceps femoris
C. Rectus femoris
D. Sartorius
E. Vastus medialis
Answer
--------------------------------------------------------------------------------

<28>An African child develops massive unilateral enlargement of his lower
face in the vicinity of the mandible. Biopsy demonstrates sheets of
medium-sized blast cells with admixed larger macrophages. This
type of tumor has been associated with which of the following?
A. Epstein-Barr virus and t(8;14)
B. Hepatitis B and t(9;22)
C. Herpesvirus and CD5
D. Human immunodeficiency virus and CD4
E. Human papillomavirus and t(2;5)
Answer
--------------------------------------------------------------------------------

<29>A 57-year-old man presents for a routine physical. His blood pressure
is 161/98 mm Hg. The patient's only complaint is that over the past
several months he has had difficulty urinating. His urine stream is
intermittent, and he has recently begun experiencing nocturia and
profound urinary urgency. Digital rectal exam reveals diffuse
enlargement of the prostate. Which of the following agents
would be most likely to effectively treat the man's urinary
tract symptoms as well as his hypertension?
A. Finasteride
B. Guanfacine
C. Hydralazine
D. Labetalol
E. Terazosin
Answer
--------------------------------------------------------------------------------

<30>A 54-year-old male with extensive, severe atherosclerosis sustains
a thrombotic occlusion of the celiac trunk. The organs that receive
their blood supply from this artery continue to function normally.
Anastomoses between which of the following pairs of arteries would
explain this phenomenon?
A. Left gastric artery and right gastric artery
B. Left gastroepiploic artery and right gastroepiploic artery
C. Proper hepatic artery and gastroduodenal artery
D. Right colic artery and middle colic artery
E. Superior pancreaticoduodenal artery and inferior pancreaticoduodenal artery
Answer
--------------------------------------------------------------------------------

<31>A cardiovascular pharmacologist is researching the effects of new
compounds on arteriolar resistance. Drug X maximally increases vascular
resistance by 50% at a dose of 20 mg/mL. Drug Y maximally increases
vascular resistance by 75% at a dose of 40 mg/mL. Which of the
following conclusions can the researcher draw from this experiment?
A. Drug X has a smaller volume of distribution than Drug Y
B. Drug X has a shorter half-life than Drug Y
C. Drug X is less efficacious than Drug Y
D. Drug X is less potent than Drug Y
E. Drug X has a lower LD50 than Drug Y
Answer
--------------------------------------------------------------------------------

<32>A 74-year-old woman, in otherwise good health, tripped and injured
her right leg 2 days previously and has been bedridden since the accident
. Two hours ago, she became delirious. On physical examination, her
temperature is 99 F, blood pressure is 120/70 mm Hg, heart rate is
110, and respiratory rate is 32. Pulse oximetry shows an oxygen
saturation of 80%, and a chest x-ray film is normal. Which of the
following is the most likely diagnosis?
A. Acute cerebral hemorrhage
B. Acute cerebral infarction
C. Myocardial infarction
D. Pulmonary infarction
E. Pulmonary thromboembolism
Answer
--------------------------------------------------------------------------------

<33>A 25-year-old pregnant woman complains to her obstetrician of dysuria.
Microscopic examination of the urine demonstrates many gram-negative rods,
many neutrophils, and a few WBC casts. Which of the following is the
specific significance of the presence of WBC casts?
A. One or both kidneys are involved in the infection
B. One or both ureters are involved in the infection
C. The bladder is involved in the infection
D. The urethra is involved in the infection
E. The uterus is involved in the infection
Answer
--------------------------------------------------------------------------------

<34>A 45-year-old female with a long history of progressive myopia develops
sudden patchy loss of vision in her right eye. She is very alarmed
and rushes to her family doctor. Funduscopic examination
reveals a large retinal detachment in the right eye.
The retina in the left eye is normal. When
the pupillary light reflex is tested by shining a light
in the right eye, the physician would likely note
A. constriction of the right pupil and constriction of the left
B. constriction of the right pupil and dilatation of the left
C. dilatation of the right pupil and constriction of the left
D. dilatation of the right pupil and dilatation of the left
E. no reaction of the right pupil and constriction of the left
Answer
--------------------------------------------------------------------------------

<35>Physical examination of a young boy reveals discharge of urine
from the umbilicus. The physician concludes that the urachus has failed
to fuse. Which of the following structures is the normal adult remnant
of the fused urachus?
A. Lateral umbilical fold
B. Medial umbilical fold
C. Medial umbilical ligament
D. Median umbilical fold
E. Median umbilical ligament
Answer
--------------------------------------------------------------------------------
<36> In this section of a healing bone fracture, which of the following
best describes the cell at the arrow?
A. Apoptotic cell
B. Bone-lining macrophage
C. Dividing osteoblast
D. Necrotic cell
E. Typical osteoblast
Answer

--------------------------------------------------------------------------------
<37>A retarded 45-year-old man living in Mexico becomes ill with pneumonia
and his family brings him across the border to the United States. He
succumbs to his illness and an autopsy is performed. Neuropathological
examination of his brain reveals neuritic plaques and neurofibrillary
tangles. Which of the following was the most likely cause of this man's
retardation?
A. Down's syndrome
B. Edwards' syndrome
C. Fragile X syndrome
D. Patau syndrome
E. Supernumerary Y syndrome


Answer
--------------------------------------------------------------------------------

<38>A 59-year-old woman presents with complaints of "feeling tired." She
also admits to frequent headaches, which occur on one side and are
throbbing. Physical examination is remarkable for a temperature of
100°F (37.8°C) and tenderness over both temples. Laboratory tests
reveal a slightly decreased hematocrit and an elevated erythrocyte
sedimentation rate. This patient should be treated aggressively to
prevent the development of
A. blindness
B. deafness
C. loss of tactile sensation
D. loss of the ability to speak
E. paralysis
Answer
--------------------------------------------------------------------------------

<39>A newborn baby has projectile vomiting shortly after each feeding. It is determined that there is obstruction of the digestive tract as a result of an annular pancreas. Annular pancreas is a result of an abnormality in which of the following processes?
A. Rotation of the dorsal pancreatic bud around the first part of the duodenum
B. Rotation of the dorsal pancreatic bud around the second part of the duodenum
C. Rotation of the dorsal pancreatic bud around the third part of the duodenum
D. Rotation of the ventral pancreatic bud around the first part of the duodenum
E. Rotation of the ventral pancreatic bud around the second part of the duodenum
Answer
--------------------------------------------------------------------------------

<40>A debilitated 72-year-old woman develops dry cough, fever, headache,
and muscular pains. She treats herself with aspirin and ampicillin without
any improvement. Her children take her to a local hospital, where chest
x-ray films reveal scattered opacities, suggestive of interstitial
infiltration. Laboratory investigations demonstrate the presence of
cold agglutinins. She is treated with erythromycin, and her symptoms
rapidly improve. Which of the following is the most likely etiologic
agent of this patient's condition?
A. Influenza virus
B. Mycoplasma pneumoniae
C. Pneumocystis carinii
D. Respiratory syncytial virus
E. Streptococcus pneumoniae
Answer
--------------------------------------------------------------------------------

<41>Which of the following hormones is most important in initiating gall
bladder contraction?
A. Cholecystokinin (CCK)
B. Gastric inhibitory peptide (GIP)
C. Gastrin
D. Secretin
E. Vasoactive intestinal polypeptide (VIP)
Answer
--------------------------------------------------------------------------------

>42>A patient complains to her family physician that "When it's time to go
to work, I just can't seem to get out of the house. I have a lot of
windows, and I need to check them all three times. Then, I can never
be sure the door is locked, so I check it 3 times. I've been late for
work a few times, but this is the only way I can be sure the house is
safe. Sometimes I think I'm going to go crazy." Which of the following
is the most likely diagnosis?
A. Adjustment disorder with anxiety
B. Agoraphobia without history of panic disorder
C. Generalized anxiety disorder
D. Obsessive compulsive disorder
E. Panic disorder with agoraphobia
Answer
--------------------------------------------------------------------------------

<43>A 32-year-old male, infected with HIV, is diagnosed with Hodgkin's
lymphoma. If the patient's CD4 count is 505/mm3, which of the following
agents would be suitable for the treatment of this patient's
lymphoma
without further compromising his immune system?
A. Busulfan
B. Cisplatin
C. Cyclophosphamide
D. Paclitaxel
E. Vincristine
Answer
--------------------------------------------------------------------------------

<44>A 62-year-old Type 2 diabetic patient presents with complaints of
malaise, myalgias, respiratory distress, and increased somnolence. If
laboratory examination reveals an anion gap of 26 mmol/L, HCO3- of 17
mmol/L and an arterial blood pH of 7.27, the patient is most likely
receiving
A. glyburide
B. metformin
C. repaglinide
D. miglitol
E. troglitazone
Answer
--------------------------------------------------------------------------------

<45>A 24-year-old delivery driver is involved in an accident and sustains
a wide abrasion over his left elbow. The abrasion results in the total
loss of epidermis over a large area of his left arm, but one month
later, the abrasion has healed, with regrowth of the epidermis.
Which of the following mechanisms accounts for the restoration
of the epidermis over the abraded area?
A. Growth of epidermis from hair follicles and sweat glands in the dermis
B. Migration of endothelial cells from newly grown capillaries
C. Transformation of dermal fibroblasts into epidermal cells
D. Transformation of macrophages into epidermal cells
E. Transformation of melanocytes into epidermal cells
Answer
--------------------------------------------------------------------------------

<46> The diagram above shows spirographic tracings of forced expirations
from two different individuals. Trace X was obtained from a person with
healthy lungs. Which of the following is most likely represented by
trace Y?
A. Asthma
B. Bronchospasm
C. Emphysema
D. Interstitial fibrosis
E. Old age
Answer

--------------------------------------------------------------------------------
<47>A patient has multiple, pearly papules on the face. Biopsy shows a
malignant tumor. Which of the following features would most likely be
seen on microscopic examination?
A. Cytoplasmic viral inclusions
B. Keratin "pearls"
C. Melanin
D. Palisading nuclei
E. S-100 positivity
Answer
--------------------------------------------------------------------------------

<48>A neonate develops marked unconjugated hyperbilirubinemia. No hemolysis
can be demonstrated and other liver function tests are normal. There is no
bilirubin found in the urine. This infant's condition continues to
deteriorate and he dies at 2 weeks of age. To which of the following
conditions did the infant most likely succumb?
A. Crigler-Najjar syndrome, Type I
B. Crigler-Najjar syndrome, Type II
C. Dubin-Johnson syndrome
D. Gilbert's syndrome
E. Rotor syndrome
Answer
--------------------------------------------------------------------------------

<49>A 41-year-old woman presents with chronic widespread musculoskeletal
pain, fatigue, and frequent headaches. She states that her musculoskeletal
pain improves slightly with exercise. On examination, painful trigger
points are produced by palpitation of the trapezius and lateral
epicondyle of the elbow. If objective signs of inflammation are
absent and laboratory studies are normal, this patient would most
likely be responsive to which of the following drugs?
A. Amitriptyline
B. Cefaclor
C. Naproxen
D. Oxycodone
E. Prednisone
Answer
--------------------------------------------------------------------------------

<50>A 29-year-old medical student developed a positive PPD (purified protein
derivative) test. She was started on isoniazid (INH) and rifampin
prophylaxis. Three months into her therapy, she began to experience
muscle fasciculations and convulsions. Administration of which of
the following vitamins might have prevented these symptoms?
A. Niacin
B. Pyridoxine
C. Riboflavin
D. Thiamine
E. Vitamin C
Answer
--------------------------------------------------------------------------------

Answers
--------------------------------------------------------------------------------

1The correct answer is C. A small percentage of patients (5% to 10%) develop a nonoliguric form of acute renal failure when treated with aminoglycosides such as gentamicin. Gentamicin can accumulate in the kidney to produce a delayed form of acute renal failure resulting in an elevation of the serum creatinine level. The nonoliguric form of renal failure, seen in this patient, is the typical presentation for gentamicin nephrotoxicity.
Acute glomerulonephritis (choice A) is typically associated with hypertension and the appearance of an active urinary sediment containing casts and red blood cells.
Cephalothin (choice B) is a first-generation cephalosporin commonly used in the treatment of severe infection of the genitourinary tract, gastrointestinal tract, and respiratory tract, as well as skin infections. This antibiotic can produce an acute interstitial nephritis; however, the patient's presentation is consistent with gentamicin nephrotoxicity. Interstitial nephritis is commonly associated with the development of acute renal failure, fever, rash, and eosinophilia.
Renal artery occlusion (choice D) is commonly caused by thrombosis or embolism. The clinical features of acute renal artery occlusion are hematuria, flank pain, fever, nausea, elevated LDH, elevated SGOT and acute renal failure.
Since the patient has normal vital signs and no history of hypotension, a diagnosis of sepsis (choice E) is unlikely.
--------------------------------------------------------------------------------2The correct answer is E. The phrase "mosaic pattern" of newly formed woven bone is a specific tip-off for Paget's disease of bone, and is not seen in other bone conditions. The clinical and radiologic presentation are typical; an increased hat size may also be a clue. In its early stages, Paget's disease is characterized by osteolysis, producing patchwork areas of bone resorption with bizarre, large osteoclasts. In the middle stage of the disease, secondary osteoblastic activity compensates with new bone formation, producing the mosaic pattern. In late Paget's, the bones are dense and osteosclerotic. Paget's disease is suspected to be related to prior viral infection, but the cause remains mysterious. Complications include myelophthisic anemia, high output cardiac failure, pain secondary to nerve compression, deformities secondary to skeletal changes, and in about 1% of patients, osteosarcoma or other sarcoma, typically involving the jaw, pelvis, or femur.
An increased incidence of astrocytomas (choice A) is associated with tuberous sclerosis.
Hodgkin's lymphoma (choice B) is usually a disease of young adults, although older patients may have the lymphocyte-depleted form.
Meningiomas (choice C) are mostly benign tumors that affect adults, especially females. There may be an association with breast cancer, possibly related to high estrogen states.
Non-Hodgkin's lymphoma (choice D) is more common in AIDS and other immunodeficiency states, although the incidence in the immunocompetent is increasing.
--------------------------------------------------------------------------------3The correct answer is C. This question required that you know two pieces of information: (1) renin is secreted from the juxtaglomerular (JG) cells; and (2) the location of the JG cells in a picture that includes both the glomerulus and the juxtaglomerular apparatus. Remember that renin is responsible for converting angiotensinogen to angiotensin I, which is subsequently converted to angiotensin II in the lungs by an important enzyme appropriately called angiotensin-converting enzyme. Angiotensin II helps preserve blood pressure both by its potent vasoconstrictor properties and by its ability to stimulate aldosterone secretion from the adrenal cortex.
The label for choice A points to the glomerular basement membrane.
The label for choice B points to the epithelium of Bowman's capsule.
The label for choice D points to the macula densa cells, which are thought to sense sodium concentration in the distal convoluted tubule.
The label for choice E points to the Polkissen cells. Their function is unknown.
--------------------------------------------------------------------------------4The correct answer is B. The findings suggest disseminated intravascular coagulation (DIC), which is a feared complication of many other disorders, such as obstetrical catastrophes, metastatic cancer, massive trauma, and bacterial sepsis. The basic defect in DIC is a coagulopathy characterized by bleeding from mucosal surfaces, thrombocytopenia, prolonged PT and PTT, decreased fibrinogen level, and elevated fibrin split products. Helmet cells and schistocytes (fragmented red blood cells) are seen on peripheral blood smear.
Autoimmune hemolysis (choice A) and hereditary spherocytosis (choice C) would be characterized by spherocytes in the peripheral smear.
Macro-ovalocytes and hypersegmented neutrophils can be seen in megaloblastic anemia (choice D).
Sickle cells are seen in sickle cell anemia (choice E).
--------------------------------------------------------------------------------5The correct answer is C. The most common form of atrial septal defect is located near the foramen ovale (not to be confused with a patent foramen ovale, which is of little or no hemodynamic significance). They result from incomplete adhesion between the septum primum and the septum secundum during development.
Atrial septal defects less commonly result from failures of formation of the septum primum (choice A) and septum secundum (choice B).
Malformations of the interventricular septum (choices D and E) cause ventricular septal defects rather than atrial septal defects.
--------------------------------------------------------------------------------6The correct answer is D. Leuprolide is a GnRH analog. Given long-term in a continuous fashion, it will inhibit FSH and LH release, thereby decreasing testosterone production and exacting a chemical castration in men. It can be used in the treatment of prostate cancer, polycystic ovary syndrome, uterine fibroids, and endometriosis.
Inhibition of 5a-reductase (choice A) is the mechanism of action of finasteride. It thereby inhibits the production of dihydrotestosterone. It is used in the treatment of benign prostatic hyperplasia (BPH).
Flutamide is another drug used in the treatment of prostate cancer. It is a competitive antagonist at androgen receptors (choice B).
Since LH activates interstitial cells to secrete testosterone, a synthetic analog of LH (choice C) would not be appropriate treatment for prostatic cancer. The same goes for a testosterone analog (choice E).
Here is a brief chart that will aid you in remembering the actions of these similar sounding drugs:
Drug Action Indication
Leuprolide GnRh analog Prostate CA
Flutamide Competitive androgen antagonist Prostate CA
Finasteride 5a-reductase inhibitor BPH
Remember, "loo"prolide and "floo"tamide are both used for prostate cancer. Finasteride is used for BPH.
--------------------------------------------------------------------------------
7The correct answer is D. 21-hydroxylase deficiency is the most common form of congenital adrenal hyperplasia. The simple virilizing variant (without salt wasting) is most common, but with severe 21-hydroxylase deficiency, virilization and salt wasting occur. The infant described above exhibits salt wasting and hyperkalemia because aldosterone secretion is diminished by the enzyme deficiency. The hypoglycemia is due to cortisol deficiency. Because cortisol secretion is diminished in congenital adrenal hyperplasia, ACTH secretion from the anterior pituitary is increased due to loss of negative feedback inhibition. The high levels of ACTH are responsible for the adrenal hyperplasia and the increased secretion of the adrenal androgens, dehydroepiandrosterone and androstenedione, which are responsible for the virilization of the external genitalia. 17-OH progesterone is the steroid precursor just proximal to 21-hydroxylase and is also increased because of the excessive drive to the adrenal cortex by ACTH.
5-alpha-reductase deficiency (choice A) in male fetuses will produce normal differentiation of the internal reproductive tracts, but the external genitalia will be feminized. This is because testosterone needs to be converted to dihydrotestosterone (by 5-alpha-reductase) in the external genitalia and the prostate for normal differentiation into the male phenotype.
11-beta-hydroxylase deficiency (choice B) is another form of congenital adrenal hyperplasia. It is characterized by salt retention due to excessive secretion by the inner zones of the adrenal cortex of the weak mineralocorticoid, deoxycorticosterone. Again, the excessive drive to the adrenal cortex is due to increased ACTH resulting from diminished negative feedback suppression by cortisol. The adrenal also secretes excessive androgens and virilization occurs in female fetuses.
17-alpha-hydroxylase deficiency (choice C) is another from of congenital adrenal hyperplasia that is accompanied by salt retention. The high levels of ACTH drive the adrenal cortex to secrete increased amounts of deoxycorticosterone and corticosterone, both of which have weak mineralocorticoid activity. Without the ability to 17-alpha-hydroxylate progesterone or pregnenolone, steroid-secreting cells cannot produce sex steroids. When 17-alpha-hydroxylase deficiency is present in the adrenal cortex, it is also present in the gonads. Hence, whether it occurs in a male or female fetus, sex steroid production will be diminished. Female fetuses will develop normal reproductive tracts and genitalia since these structures are programmed in utero to "automatically" become female. Male fetuses, however, will have their reproductive tracts and genitalia feminized.
Complete androgen resistance (choice E) results in feminization of affected male fetuses. It is characterized by an XY genotypic male with phenotypically female external genitalia and a vagina that ends as a blind sac.
--------------------------------------------------------------------------------8The correct answer is E. Vibrio vulnificus is an extremely invasive organism, producing a septicemia in patients after eating raw shellfish, or causing wound infections, cellulitis, fasciitis, and myositis after exposure to seawater or after cleaning shellfish. Patients at high risk for septicemia include those with liver disease, congestive heart failure, diabetes mellitus, renal failure, hemochromatosis, and immunosuppression.
Citrobacter diversus (choice A) produces neonatal meningitis and can be frequently cultured from the umbilicus.
Enterotoxigenic E. coli (choice B) produces the classic traveler's diarrhea. The toxin is ingested in water and salads. The incubation period is approximately 12 hours. The diarrhea is non-inflammatory and treatment is supportive.
Providencia stuartii (choice C) is a gram-negative rod related to Proteus. It is a common cause of nosocomial bacteremia in nursing home patients with chronic catheterization.
Vibrio cholerae (choice D) produces a non-invasive, non-inflammatory, high-volume secretory diarrhea that is toxin-mediated.
--------------------------------------------------------------------------------9The correct answer is D. The disease in question is leprosy, or Hansen's disease. A key feature in the description is the fact that the organism is acid-fast. Both of the mycobacteria, M. avium-intracellulare and M. leprae are strongly acid-fast, that is they retain the carbol fuchsin dye in the face of acid-alcohol decolorization. M. leprae has a predilection for the skin and cutaneous nerves, thereby producing the symptoms of depigmentation and anesthetic cutaneous lesions. This loss of peripheral nerve function leads to many of the disfiguring features of the disease; because the patients do not have normal pain sensation, they sustain repeated injuries. In addition, the organism attacks cartilage and causes granuloma formation in the skin, leading to some of the facial disfigurement.
Bartonella henselae (choice A) is a very small, gram-negative bacterium that is closely related to the rickettsia, although it is able to be cultured on lifeless media. It is the cause of cat-scratch disease, a local, chronic lymphadenitis most commonly seen in children, and bacillary angiomatosis, a disease seen particularly in AIDS patients.
Listeria monocytogenes (choice B) is a ubiquitous microbe that causes disease in over 100 animal species. Although it is best known as an agent of meningitis in the newborn, it is a cause of multiple other diseases. A characteristic feature of these infections is the development of granulomas at the site of the infection. The organism is not acid-fast and has no particular predilection for skin or nervous tissues.
M. avium-intracellulare (choice C) causes tuberculosis-like pulmonary disease in the immunosuppressed.
Nocardia asteroides (choice E) primarily produces pulmonary infections in humans. The organism is consider to be "weakly" acid-fast, meaning that if the amount of HCl used in the decolorization step is reduced, the organisms will retain the carbolfuchsin primary stain.
--------------------------------------------------------------------------------10The correct answer is A. The cells described are the primordial eggs, which remain stopped in the diplotene stage of the first meiotic division from before birth until fertilization, a period which may be 40 or more years.
Choices B and E are incorrect because the oocytes are stopped in the first, not second meiotic division.
Choices C and D are incorrect because the cells described are oocytes and are not in mitosis.
--------------------------------------------------------------------------------11The correct answer is D. This is impetigo, which is typically seen in preschool children with poor hygiene, particularly in the summer in warm climates. The characteristic lesion has a large golden crust. Most cases are caused by Staphylococcus aureus; Streptococcus pyogenes is occasionally implicated. Impetigo is highly infectious, and mini-epidemics can occur in daycare settings. The initial treatment is typically with penicillins and topical preparations. Methicillin-resistant strains are presently rare in this setting, but can occur.
Aphthous ulcers (choice A), commonly known as "canker sores," are painful, shallow ulcers of the oral cavity.
Erysipelas (choice B) is a different type of skin infection, often caused by Streptococcus pyogenes (also sometimes others including Staphylococcus), and is characterized by large erythematous patches.
Herpes simplex I (choice C) causes tiny oral and perioral vesicles, but not large golden crusts.
Measles (choice E) causes a blotchy erythematous rash.
--------------------------------------------------------------------------------12The correct answer is C. This little boy has a Meckel's diverticulum, an ileal outpocketing typically located within 50-75 cm of the ileocecal valve. It is a congenital anomaly resulting from the persistence of the vitelline (omphalomesenteric) duct. Approximately half cause ulceration, inflammation, and gastrointestinal bleeding due to the presence of ectopic acid-secreting gastric epithelium. Pancreatic tissue may sometimes occur in these diverticula as well. Note that this is the most common type of congenital gastrointestinal anomaly.
Something else to keep in mind: A favorite question attendings ask on the wards is the rule of 2's associated with Meckel's diverticulum: it occurs in about 2% of children, occurs within approximately 2 feet of the ileocecal valve, contains 2 types of ectopic mucosa (gastric and pancreatic), and its symptoms usually occur by age 2.
All of the other answer choices have no relationship to Meckel's diverticulum.
--------------------------------------------------------------------------------13The correct answer is B. The patient has primary atypical pneumonia caused by Mycoplasma pneumoniae. This organism is fastidious and difficult to culture in the laboratory, however serodiagnosis can be most helpful. Patients typically produce one or two heterophile antibodies during the course of the infection; one agglutinates human O+ RBCs in the cold (the cold hemagglutinin) while the other causes the agglutination of a strain of Streptococcus salivarius termed strain MG (the Strep MG agglutinins).
Klebsiella pneumoniae (choice A) is readily cultured on routine laboratory media and characteristically produces pneumonia with blood clots in the sputum (red currant jelly sputum), which may be indicative of pulmonary abscess development.
Parainfluenza viruses (choice C) cause croup, which is characterized by a dry, "barking" cough. It is more of a tracheitis, bronchitis, and bronchiolitis than a pneumonitis. No heterophile antibodies are produced in these patients.
Respiratory syncytial virus (choice D) causes an atypical pneumonitis in infants. It is usually diagnosed by the observation of syncytial masses in respiratory secretions. Cold hemagglutinins and Strep MG agglutinins are absent.
Streptococcus pneumoniae (choice E) is the number one cause of pneumonia in adults. It also causes septicemia and meningitis in the elderly. The patient has a classical acute pneumonia with a productive cough, high fever with chills, leukocytosis, tachycardia, rapid respirations and other signs of serious respiratory disease. A vaccine, composed of the capsular carbohydrate of 23 serotypes of this organism, is routinely given to individuals over the age of 60, as well as to individuals with splenic abnormalities (e.g., sickle cell disease) who are at increased risk for pneumococcal sepsis.
--------------------------------------------------------------------------------14The correct answer is F. Aneurysm of the posterior communicating artery is the second most common aneurysm of the circle of Willis (anterior communicating artery is most common) and can result in third cranial nerve palsy (paralysis). The oculomotor nerve (CN III) innervates the levator palpebrae muscle. CN III paralysis would therefore result in ptosis (drooping of the upper eyelid). CN III also innervates all of the extraocular muscles, except for the superior oblique (CN IV) and the lateral rectus muscles (CNVI). Thus, CN III palsy would result in unopposed action of the superior oblique and lateral rectus muscles, causing the affected eye to look down and out. CN III also supplies parasympathetic innervation to the sphincter muscle of the iris (which constricts the pupil) and to the ciliary muscle. Interruption of this pathway leads to a dilated and fixed pupil and to paralysis of accommodation.
Note that this question teaches you about another Boards-favorite pathology: subarachnoid hemorrhage (SAH). (In this case, it was due to rupture of a posterior communicating artery aneurysm). A classic clue to the diagnosis is a patient presenting with "the worst headache of their life." When you are presented a case of sudden severe headache, SAH should rank highly on your differential diagnosis list.
The anterior cerebral artery (choice A) supplies the medial surface of the cerebral hemisphere, from the frontal pole to the parieto-occipital sulcus. Occlusion may produce hypesthesia and paresis of the contralateral lower extremity.
The anterior choroidal artery (choice B) arises from the internal carotid artery and is not part of the circle of Willis. It perfuses the lateral ventricular choroid plexus, the hippocampus, parts of the globus pallidus and posterior limb of the internal capsule.
The anterior communicating artery (choice C) connects the two anterior cerebral arteries. It is the most common site of aneurysm in the circle of Willis and may cause aphasia, abulia (impaired initiative), and hemiparesis.
The middle cerebral artery (choice D) supplies the lateral convexity of the cerebral hemisphere, including Broca's and Wernicke's speech areas and the face and arm areas of the motor and sensory cortices. It also gives rise to the lateral striate arteries, which supply the internal capsule, caudate, putamen, and globus pallidus. The middle cerebral artery is the most common site of stroke.
The ophthalmic artery (choice E) enters the orbit with the optic nerve (CN II) and gives rise to the central artery of the retina. Occlusion results in blindness.
The posterior inferior cerebellar artery (choice G) supplies the dorsolateral medulla and the inferior surface of the cerebellar vermis. Occlusion may result in Wallenberg's syndrome: cerebellar ataxia, hypotonia, loss of pain and temperature sensation of the ipsilateral face, absence of corneal reflex ipsilaterally, contralateral loss of pain and temperature sensation in the limbs and trunk, nystagmus, ipsilateral Horner's syndrome, dysphagia, and dysphonia.
--------------------------------------------------------------------------------15The correct answer is C. A variety of glycogen storage diseases exist, corresponding to defects in different enzymes in glycogen metabolism; most of these involve the liver. McArdle's disease (Type V glycogen storage disease), due to a defect in muscle phosphorylase, is restricted to skeletal muscle. The presentation described in the question stem is typical. Many affected individuals also experience myoglobinuria. Definitive diagnosis is based on demonstration of myophosphorylase deficiency.
Hartnup's disease (choice A) is a disorder of amino acid transport.
Krabbe's disease (choice B) is a lysosomal storage disease.
Niemann-Pick disease (choice D) is a lysosomal storage disease.
Von Gierke's disease (choice E) is a glycogen storage disease with prominent involvement of liver, intestine, and kidney.
--------------------------------------------------------------------------------16The correct answer is D. The disease is Down syndrome (trisomy 21). In addition to mental retardation and the characteristic physical findings described in the question stem, duodenal atresia is fairly common, as evidenced by the "double bubble" sign on x-ray. These children are also likely to have various cardiac anomalies; endocardial cushion defect is the most common.
Atrial septal defect (choice A) is one of the most common genetic defects in the general population, but is less common than endocardial cushion defect in patients with Down syndrome.
Berry aneurysms (choice B), also known as saccular aneurysms, are typically located in the circle of Willis on the ventral surface of the brain. They occur more frequently in patients with adult polycystic disease. Rupture can produce subarachnoid hemorrhage.
Coarctation of the aorta (choice C) occurs more commonly in females with a 45, XO genotype (Turner syndrome).
Tetralogy of Fallot (choice E) is the most common cause of early cyanosis, consisting of a ventricular septal defect, right ventricular outflow tract obstruction, an overriding aorta, and right ventricular hypertrophy.
--------------------------------------------------------------------------------17The correct answer is E. All of the laboratory data in this pregnant woman are normal, hence no further study is necessary. In a normal pregnancy, both the plasma volume and RBC mass are increased with a greater increase in the plasma volume than RBC mass (2:1 ratio). This has a dilutional effect on many laboratory tests.
Increasing plasma volume in pregnancy increases the creatinine clearance (choice A) due to the expected elevation in the glomerular filtration rate (GFR). The reference intervals for serum blood urea nitrogen and creatinine are lower than normal, due to the dilutional effect of increased plasma volume and increased clearance of both analytes in the urine caused by the rise in the GFR.
The threshold for glucose is reduced in pregnancy, so patients can have a positive dipstick test for glucose in the presence of a normal serum glucose. Therefore, an oral glucose tolerance test (choice B) is not indicated.
The hemoglobin (Hb) concentration in pregnancy is normally decreased because of the dilutional effect of increased plasma volume. Since the Hb is normal (for a pregnant woman) in this patient, a serum ferritin (choice C) to rule out iron deficiency is unnecessary. Furthermore, iron deficiency is usually associated with a low MCV (microcytic anemia), and her MCV is normal.
Although sickle disease is the most common genetic hemoglobinopathy among African Americans, the patient is not anemic, so there is no reason to order a sickle cell preparation (choice D).
--------------------------------------------------------------------------------18The correct answer is D. The endometrial phase with small glands is the proliferative phase; the one with large glands with secretory cells is the secretory phase. Estrogen (choice C) is necessary for both phases, but it is the addition of progesterone (choice D), secreted by the corpus luteum after the Graafian follicle ruptures, that triggers the switch from proliferative to secretory endometrium.
Glucocorticoids (choice B) and the mineralocorticoid aldosterone (choice A) are secreted by the adrenal glands. They do not produce the endometrial changes described.
Thyroxine (choice E) is secreted by the thyroid gland, and is unrelated to the observed morphologic changes in the endometrium.
--------------------------------------------------------------------------------19The correct answer is D. The distinctive cell balls described are broken-off papillary clusters, and are considered pathognomic for papillary carcinoma of the thyroid. This is the most common form of thyroid carcinoma. It tends to present in the 3rd to 5th decade and shows a modest female predominance. Despite its propensity for local lymphatic intrusion (which may cause multifocality of tumor in the thyroid or cervical lymph node metastases), the tumor generally has an excellent prognosis with 90% 20-year survival.
Follicular carcinoma (choice A) is characterized by follicular cells and colloid on aspiration, and cannot be reliably distinguished from thyroid adenoma.
Distinctive features of Hashimoto's disease (choice B) on aspiration are lymphocytes, plasma cells, and macrophages.
The most distinctive feature of medullary carcinoma (choice C) on aspiration is the presence of amyloid.
Thyroid adenoma (choice E) shows follicular cells and colloid on aspiration, and cannot be reliably distinguished from follicular carcinoma.
--------------------------------------------------------------------------------20The correct answer is D. This is a straightforward question relating to the definition of Nissl bodies. Rough endoplasmic reticulum present in neurons are called Nissl bodies. They stain intensely with basic dyes and are found in the cell body and proximal dendrites, but not in the axon hillock or axon.
--------------------------------------------------------------------------------21The correct answer is A. Sawtooth waves appearing in bursts are associated with REM sleep.
Stage 1 (choice B) is associated with 4-7 Hz theta waves.
Stage 2 (choice C) is associated with 12-14 Hz sleep spindles and K-complexes.
Stage 3 (choice D) is associated with < 4 Hz, high-amplitude delta waves.
Stage 4 (choice E) is characterized by an EEG composed of about 50% delta waves.
Note that beta waves (15-18 Hz) occur during periods of more intense mental activity while awake. Alpha waves (8-12 Hz) occur during awake, relaxed states. REM is the stage of sleep that most resembles the awake state on the EEG.
--------------------------------------------------------------------------------22The correct answer is E. The disease is poliomyelitis. Most infections with poliovirus cause only the influenza-like symptoms, but a small percentage progress to paralytic poliomyelitis. The most common causes of death are aspiration and airway obstruction as a result of bulbar paralysis and paralysis of respiratory muscles. Arrhythmias can also be life-threatening.
Acute renal failure (choice A) is usually not seen in poliomyelitis, although the bladder may become paralyzed.
Bowel paralysis (choice B) can be seen, but is not usually life-threatening.
Fulminant liver failure (choice C) is not a feature of poliomyelitis.
Gastrointestinal bleeding (choice D) can be seen in poliomyelitis, but is not usually life-threatening.
--------------------------------------------------------------------------------23The correct answer is D. The ratio of cells in bone marrow developing along myeloid lines to cells developing along erythroid lines is 3:1. An alternative way to remember the normal marrow composition is that it typically contains about 60% granulocytes and their precursors; 20% erythroid precursors; 10% lymphocytes, monocytes, and their precursors; and 10% unidentified or disintegrating cells. These numbers are worth remembering, because shifts away from normal values may be a subtle clue to marrow abnormalities.
--------------------------------------------------------------------------------24The correct answer is E. An obese adult with glucosuria, but not ketonuria, likely has type 2 diabetes mellitus. Type 2 diabetes is characterized by insulin resistance resulting in hyperglycemia and increased serum osmolarity. The dehydration associated with osmotic diuresis makes the hyperosmolarity worse. As the osmolarity increases above 330 mOsm/L, the osmotic loss of water from neurons is sufficient to produce coma. In nonketotic, hyperosmolar coma, blood glucose values can range from 800 to 2,400 mg/dL and produce serum osmolarities of 330-440 mOsm/L.
Since type 2 diabetes is due to insulin resistance, plasma levels of insulin are usually normal to increased. Because b cells secrete insulin and C-peptide in a 1:1 ratio, plasma concentration of C-peptide would also be normal to increased (not decreased, choice A) in type 2 diabetes.
Even small amounts of insulin are sufficient to prevent ketosis. In type 2 diabetes there is enough insulin effect to prevent significant lipolysis and subsequent formation of excess ketone bodies. Hence, acidosis (choice B) is not typically associated with this disorder. Since b-hydroxybutyrate is a ketone body, its concentration in plasma is not likely to be increased (choice D).
While there is a significant autoimmune component to type 1 diabetes, type 2 diabetes is not associated with increased circulating antibodies (choice C) against b cell proteins such as glutamic acid decarboxylase.
--------------------------------------------------------------------------------25The correct answer is B. Neuromuscular development is sufficient to allow fetal movement in the eighth week of life. Other features of week 8 include the first appearance of a thin skin, a head as large as the rest of the body, forward-looking eyes, appearance of digits on the hands and feet, appearance of testes and ovaries (but not distinguishable external genitalia), and a crown-rump length of approximately 30 mm. By the end of the eighth week, nearly all adult structures have at least begun to develop, and the fetus "looks like a baby".
--------------------------------------------------------------------------------26The correct answer is A. The Rb gene is an example of a tumor suppressor gene. Tumor suppressor genes encode proteins that downregulate cell growth; consequently, their deletion leads to the development of cells with a growth advantage over normal cells. Even if you know nothing about the Rb protein, choice A is still the only logical answer because it is the only example of a protein that, if absent, would favor cell growth. The Rb protein binds to transcription factors in the nucleus, preventing cells from progressing from the S1 to M stages of the cell cycle. Children born with a 13q14 deletion have only one chromosome encoding Rb; therefore only a single "hit " is required to completely knock out Rb production and lead to the development of retinoblastoma. All of the incorrect choices are proteins encoded by oncogenes, rather than tumor suppressor genes. Oncogenes favor tumorigenesis through overexpression, not deletion.
Growth factors (choice B) are oncoproteins that are produced by tumors and have a positive feedback effect. Examples of growth factors are PDGF and fibroblast growth factor; the oncogenes encoding them are sis and hst-1, respectively.
The prototypical growth factor-binding protein (choice C) is ras, which is mutated in a large variety of cancers. Ras normally functions as an activator of protein kinases that regulate cell growth. Overactivity of the ras protein is highly mitogenic.
Growth factor receptors (choice D) are either expressed as mutant forms or overexpressed in tumors, leading to upregulation of growth. An example of a growth factor receptor oncogene is erb-B2, present in some breast cancers.
Transcription activators (choice E) are DNA-binding proteins that promote DNA transcription. Amplification of these oncogenes causes cancer by promoting the transcription of growth-related genes.
--------------------------------------------------------------------------------27The correct answer is A. The obturator nerve innervates the muscles of the medial compartment of the thigh. These include the adductor longus, adductor brevis, adductor magnus, and gracilis muscles. The adductor magnus is also innervated by the tibial nerve.
The biceps femoris (choice B) is in the posterior compartment of the thigh. The long head of the biceps femoris is innervated by the tibial portion of the sciatic nerve, and the short head of the biceps femoris is innervated by the common peroneal portion of the sciatic nerve.
The rectus femoris (choice C) and vastus medialis (choice E) are two of the four heads of the quadriceps femoris muscle. All four heads of the quadriceps femoris muscle are in the anterior compartment of the thigh, and are innervated by the femoral nerve.
The sartorius muscle (choice D) is in the anterior compartment of the thigh, and is innervated by the femoral nerve.
--------------------------------------------------------------------------------
28The correct answer is A. The patient has Burkitt's lymphoma. This type of lymphoma is a high-grade B-cell lymphoma that occurs in endemic form in Africa (it is the most common neoplasm in children in an equatorial belt that includes Africa and New Guinea) and sporadically in the United States and Europe. The sporadic form is often in an abdominal site and occurs in young adults. The African form of Burkitt's lymphoma has been strongly associated with antibodies directed against Epstein-Barr virus; the association is weaker in sporadic cases. A characteristic translocation, t(8;14) (q24.l3;q32.33) has been described.
Hepatitis B (choice B) is associated with hepatocellular carcinoma. t(9;22) is the Philadelphia chromosome, which is seen in some cases of CML and AML.
Herpesvirus (choice C) does not have a strong tumor association, although a link to cervical cancer has intermittently been proposed. CD5 is a marker seen in small lymphocytic and mantle cell lymphomas.
HIV (choice D) is linked to Kaposi's sarcoma (and AIDS). Some patients also develop primary lymphomas (not usually Burkitt's). CD4 is a marker for helper T cells and some T cell lymphomas.
Human papillomavirus (choice E) is linked with common warts, genital condylomata, and genital cancers. t(2;5) is linked to anaplastic large cell lymphoma.
--------------------------------------------------------------------------------29The correct answer is E. The patient is presenting with hypertension and signs and symptoms of benign prostatic hyperplasia (BPH). The essential diagnostic characteristics of BPH include a decrease in the force and caliber of the urinary stream, nocturia, high post-void residual volume, urinary retention, and azotemia. Terazosi

usmle-mcq's-9

<1>A 61-year-old woman with leukemia abruptly develops an intensely
itchy rash. Physical examination demonstrates multiple erythematous patches
of the distal arms and legs, some of which involve the palms and soles.
Some of the patches show central clearing with surrounding erythematous
rings. Which of the following is the most likely diagnosis?
A. Erythema migrans chronicum
B. Erythema multiforme
C. Kaposi's sarcoma
D. Psoriasis
E. Urticaria
Answer
--------------------------------------------------------------------------------

<2>A 52-year-old woman presents to her physician for a check-up. She is
recovering from a wrist fracture after a fall. Dual energy x-ray
absorptiometry of the hip had shown her to have osteoporosis. She
became menopausal at age 50 and did not begin hormone replacement
therapy because of a strong family history of breast cancer. She
now fears a future hip fracture and would like to begin a bone
loss prevention regime.Which of the following pharmaceutical
agents is most appropriate for this patient?
A. Calcitonin nasal spray
B. Oral conjugated estrogen
C. Raloxifene
D. Tamoxifen
E. Transdermal estradiol
Answer
--------------------------------------------------------------------------------

<3>A fracture of the humerus neck damages a nerve running around the
humeral neck. After the fracture heals, the patient notices a marked
decrease in his ability to raise his arm over his head. The denervated
muscle arises from which of the following?
A. Acromion alone
B. Clavicle and acromion
C. Clavicle and coracoid
D. Coracoid alone
E. Coracoid and acromion
Answer
--------------------------------------------------------------------------------

<4>A patient is admitted to a psychiatric hospital after having been picked
up by the police for making inappropriate sexual advances. A detailed
psychiatric interview demonstrates deficits in memory, insight,
judgement, personal appearance, and social behavior. The patient
is witnessed experiencing a possible epileptic seizure. Over a
period of several years, motor findings also develop, including
relaxed, but expressionless facies, tremor, dysarthria,
and pupillary abnormalities. Which of the following tests performed on his cerebrospinal fluid would most likely be diagnostic?
A. CSF glucose
B. FTA-ABS
C. Gram's stain
D. Lymphocyte count
E. Neutrophil count
Answer
--------------------------------------------------------------------------------

5>A 29-year-old single man seeks psychiatric treatment to help him deal
with difficulties in his personal life. Although the man is a successful
computer programmer, he feels unsatisfied with his interpersonal
relationships. He reports being attracted to several of his female
coworkers, but is "too shy" to talk to them about anything other
than superficial subjects such as the weather. He would like to ask
one of the women out on a date, but is afraid of being rejected.
Which of the following diagnoses is most appropriate?
A. Avoidant personality disorder
B. Borderline personality disorder
C. Dependent personality disorder
D. Narcissistic personality disorder
E. Schizotypal personality disorder
Answer
--------------------------------------------------------------------------------


<6>A 3-month-old infant presents with a 3-day history of fever, cough,
and poor feeding. On examination, the baby appears ill and has a
temperature of 102 F and a respiratory rate of 32. A chest x-ray
film shows bilateral patchy infiltrates in the lungs. Which of
the following is the most likely etiologic agent?
A. Coronavirus
B. Influenza type A
C. Parainfluenza type 1
D. Respiratory syncytial virus
E. Rhinovirus
Answer
--------------------------------------------------------------------------------

<7>A 35-year-old male presents to the urologist for an infertility
evaluation. A biopsy of his testis is performed to check sperm
production and maturation. A microscopic section reveals only
a few germ cells near the basal lamina in the seminiferous
tubule. Which of the following cells is the germ cell closest
to the basal lamina in the seminiferous tubule?
A. Primary spermatocyte
B. Secondary spermatocyte
C. Spermatid
D. Spermatogonia
E. Spermatozoa
Answer
--------------------------------------------------------------------------------

<8>A 30-year-old otherwise healthy woman presents to her physician
with complaints of fatigue and dyspnea. Physical examination reveals
normal breath sounds and the presence of third and fourth heart sounds.
Chest x-ray shows clear lung fields but right ventricular enlargement,
main pulmonary artery enlargement, and "pruning" of the peripheral
vasculature. Electrocardiogram shows right axis deviation and right
ventricular hypertrophy. Left ventricular function appears normal
on echocardiography. Serologic studies show antinuclear antibodies.
Which of the following pathological findings would this patient
also show, either at autopsy or if an appropriate biopsy was taken?
A. Mural thrombus of the right atrium
B. Necrosis and scarring of the left ventricle
C. Plexogenic pulmonary vasculopathy
D. Pulmonary artery stenosis
E. Severe pulmonary fibrosis
Answer
--------------------------------------------------------------------------------

<9>During an abdominal surgical procedure, the surgeon wishes to locate
the ureter in order to ensure that it is not injured. The ureter may be
found immediately anterior to the origin of the
A. common iliac artery
B. external iliac artery
C. internal iliac artery
D. gonadal artery
E. renal artery
Answer
--------------------------------------------------------------------------------

<10>A patient with mild congestive heart failure is treated with high-dose
furosemide and diureses 25 pounds of fluid. A complete blood count (CBC)
taken before the diuresis shows an RBC count of 4 million/mm3; a CBC
taken after diuresis shows a RBC count of 7 million/mm3. Which of the
following is the most likely explanation?
A. Cyanotic heart disease
B. Increased erythropoietin
C. Polycythemia vera
D. Relative polycythemia
E. Renal cell carcinoma
Answer
--------------------------------------------------------------------------------

<11>A newborn male child is noted to have hypospadias. A complete
evaluation determines that the child has no other genitourinary
anomalies. Nonetheless, hypospadias repair will be performed to
prevent which of the following possible sequelae?
A. Bladder exstrophy
B. Hydrocele
C. Phimosis
D. Urachal cysts
E. Urinary tract infection
Answer
--------------------------------------------------------------------------------

<12>During the passage of an intravenous catheter, numerous endothelial
cells are dislodged from the lining of the popliteal vein. What substance
allows platelet adhesion to the exposed collagen fibers?
A. Factor VIII
B. Factor IX
C. Fibronectin
D. Tissue factor
E. Von Willebrand factor
Answer
--------------------------------------------------------------------------------

<13>A 57-year-old man presents to the emergency department with a
nosebleed for the past 2 hours. The patient received a prosthetic
heart valve 5 months ago and is currently taking warfarin (7.5 mg per day)
and oral antibiotics. Laboratory evaluation reveals
an INR (international normalized ratio, the ratio of
patient to normal prothrombin times) of 6.4. Which of
the following antibiotics is the patient most likely taking?
A. Ampicillin
B. Cephalexin
C. Nitrofurantoin
D. Norfloxacin
E. Phenazopyridine
Answer
--------------------------------------------------------------------------------

<14>A 47-year-old woman presents with complaints of nervousness and
increased sensitivity to hot weather. She is diagnosed with
hyperthyroidism and prescribed propylthiouracil. What is
the principal mechanism by which this drug acts?
A. Decreasing the efficacy of TSH binding to the thyroid TSH receptor
B. Decreasing the rate of proteolysis of thyroglobulin
C. Increasing the amount of 3,3',5'-triiodothyronine (reverse T3; rT3)
D. Inhibiting deiodination of thyroxine (T4)
E. Inhibiting the uptake of iodide into the thyroid gland
Answer
--------------------------------------------------------------------------------

<15>Which of the following neurotransmitters is most important for the
induction of REMsleep?
A. Acetylcholine
B. Dopamine
C. Epinephrine
D. Norepinephrine
E. Serotonin
Answer
--------------------------------------------------------------------------------

<16>A 37-year-old woman presents with 3 days of progressive joint pain
in her ankles, knees, and wrists. She had three similar episodes
over the past several years. On examination, she has a temperature
of 38.7 C, her blood pressure is 110/70 mm Hg, and her heart rate
is 90/min. She has a diffuse petechial rash over her trunk and extensor
surfaces. Her ankles and knees are swollen, red, and tender with
decreased range of motion, and there is tenderness over the tendon
sheaths of her hands and forearms. Blood cultures are negative.
Aspiration of joint fluid reveals a white cell count of
22,000/mm3 with no visible organisms, but culture on
chocolate agar is positive. Which of the following
is an attribute of the causative organism that allows
it to produce recurrent infections?
A. It is an intracellular pathogen
B. It is resistant to ceftriaxone
C. It is resistant to complement-mediated lysis
D. Its capsule is not immunogenic
E. Its pili undergo antigenic and phase variation
Answer
--------------------------------------------------------------------------------

<17>The data presented below compares the results of a diagnostic
test in the presence and absence of a disease.
Disease present Disease absent
Positive test 40 5
Negative test 10 95
Total 50 100
The specificity of the test is
A. 0.05
B. 0.40
C. 0.80
D. 0.90
E. 0.95
Answer
--------------------------------------------------------------------------------

<18>Blood stored in a blood bank tends, with time, to become relatively depleted of 2,3-diphosphoglycerate. What effect does this have on the hemoglobin-oxygen dissociation curve?
A. Shifts the curve to the left, so that the hemoglobin has a decreased oxygen affinity
B. Shifts the curve to the left, so that the hemoglobin has an increased oxygen affinity
C. Shifts the curve to the right, so that the hemoglobin has a decreased oxygen affinity
D. Shifts the curve to the right, so that the hemoglobin has an increased oxygen affinity
E. Does not change the dissociation curve
Answer
--------------------------------------------------------------------------------

<19>A psychotic, indigent man with a history of multisubstance
abuse has been involuntarily hospitalized for 1 week. Because of
persistent diarrhea, stools are sent for ova and parasites,
revealing numerous granular, spherical, thin-walled cysts
measuring 10-20 mm in diameter. Trichrome stains show up to
four nuclei in most of the cysts. These finding are consistent
with an infection by which of the following organisms?
A. Cryptosporidium parvum
B. Dientamoeba fragilis
C. Entamoeba histolytica
D. Giardia lamblia
E. Isospora belli
Answer
--------------------------------------------------------------------------------

<20>A child who understands that the volume of a liquid poured out of
a narrow glass remains the same when poured into a wider glass is at
which of Piaget's stages of intellectual development?
A. Concrete operations
B. Formal operations
C. Preoperational
D. Sensorimotor
Answer
--------------------------------------------------------------------------------

<21>A 4-year-old girl is brought to a specialty clinic by her foster parents because of a limp in her right leg and a serum alkaline phosphatase (ALP) that is 10 times the adult upper limit of normal for the test. The child is withdrawn and clinging to her foster mother. Physical examination is remarkable for a blue-green discoloration of the skin overlying a tender, 3-cm mass on the anterior portion of the right mid-thigh. An x-ray of the right thigh reveals a dense mass within the femur that extends into the surrounding tissue. Which of the following best describes the increased alkaline phosphatase and physical findings in this case?
A. ALP is abnormal for her age and the bone lesion represents an area of aseptic necrosis
B. ALP is abnormal for her age and the bone lesion represents repair of a femoral fracture
C. ALP is abnormal for her age and is unrelated to the bone lesion
D. ALP is normal for her age and the bone lesion is an osteogenic sarcoma
E. ALP is normal for her age and is unrelated to the bone lesion
Answer
--------------------------------------------------------------------------------

<22>Which of the following drugs antagonizes both the vascular and cardiac
actions of norepinephrine?
A. Atenolol
B. Esmolol
C. Labetalol
D. Metaproterenol
E. Prazosin
Answer
--------------------------------------------------------------------------------

<23>During surgery to treat an ischioanal abscess, the inferior rectal
nerve is damaged. This nerve is a direct branch of which of the following
nerves?
A. Inferior gluteal nerve
B. Pelvic splanchnic nerve
C. Pudendal nerve
D. Sciatic nerve
E. Superior gluteal nerve
Answer
--------------------------------------------------------------------------------

<24>A 15-year-old girl is evaluated for failure to begin menstruation.
Physical examination demonstrates short stature and a webbed neck.
Chromosomal analysis demonstrates a lack of one X chromosome. This
patient should be specifically evaluated for which of the following
cardiovascular anomalies?
A. Coarctation of the aorta
B. Dextrocardia
C. Ostium primum septal defect
D. Pulmonary stenosis
E. Tetralogy of Fallot
Answer
--------------------------------------------------------------------------------

<25>A baby is born with a testicular mass. Histologic sections made of the
homogeneous yellow-white mass after its removal demonstrate
epithelial-lined spaces that have flattened-to-cuboidal epithelial
cells with vacuolated cytoplasm containing eosinophilic, hyaline-like
globules. Scattered structures resembling primitive glomeruli
(endodermal sinuses) are also seen. If appropriate
immunohistochemical stains are performed, the eosinophilic
cytoplasmic globules would most likely contain which of the following?
A. Alpha-fetoprotein
B. Estrogen receptors
C. Human chorionic gonadotropin
D. Human papilloma virus
E. Melanin
Answer
--------------------------------------------------------------------------------

<26>A surgeon wishes to perform a splenectomy on a patient who has
been in an automobile accident. Before removing the spleen, the
splenic artery and splenic vein are ligated. Within which of the
following peritoneal structures are the splenic artery and vein found?
A. Gastrocolic ligament
B. Gastrosplenic ligament
C. Lesser omentum
D. Splenorenal ligament
Answer
--------------------------------------------------------------------------------

<27>A 65-year-old woman with a long-standing disease has bone marrow
fibrosis and increased bone remodeling, with bone resorption exceeding
bone formation. She has a history of passing calcium-oxalate kidney
stones. Which of the following lab result profiles would be expected
in the serum of this patient?
Calcium Phosphate PTH
A. decreased decreased increased
B. decreased increased decreased
C. decreased increased increased
D. increased decreased increased
E. increased increased increased
Answer
--------------------------------------------------------------------------------

<28>Upon examination of a newborn male, a cystic structure is found
in the scrotum. It is determined that there is a hydrocele of the
spermatic cord. Which of the following is the most likely etiology
of this finding?
A. Communication between the epididymis and the tunica vaginalis
B. Failure of the processus vaginalis to form
C. Failure of the processus vaginalis to fuse
D. Incomplete fusion of the processus vaginalis
E. Varicosities of the spermatic vein
Answer
--------------------------------------------------------------------------------

<29>A Brazilian immigrant is hired at a meat-packing plant, and undergoes
an employment physical. Chest x-ray demonstrates a patchy, bilateral
pneumonia and a lung mass, and he is referred to a specialist. Biopsy
of the mass demonstrates fungal organisms with a few very distinctive
"pilot's wheel" yeast forms. Which of the following is the most likely
diagnosis?
A. Blastomycosis
B. Coccidioidomycosis
C. Histoplasmosis
D. Paracoccidioidomycosis
E. Sporotrichosis
Answer
--------------------------------------------------------------------------------

<30>A 34-year-old female is brought to the emergency room with severe
muscle cramps and carpopedal spasms. The patient was noted to be extremely
irritable and was complaining of tingling around the mouth and in the hands
and feet. A few hours later, laboratory examination reveals
sodium 140 mEq/L, potassium 4.2 mEq/L, chloride 101 mEq/L,
calcium 6.4 mg/dL, phosphate 5.1 mg/dL,
magnesium 2.4 (normal 1.8-3.8 mg/dL) and alkaline phosphatase 67 U/L. A CT scan of the head shows basal ganglia calcifications. Prolonged QT intervals and T wave abnormalities are noted on electrocardiogram. The history is pertinent for a thyroidectomy two months prior to admission. Which of the following conclusions is most consistent with these data?
A. An increase in dietary vitamin D is warrante
B. Hepatocytes have a low 25-hydroxylase activity
C. Intestinal cells are underexpressing calcium transporter genes
D. Isolated cells from the kidney have high 1-hydroxylase activity
E. The levels of 1,25-dihydroxy vitamin D are normal
Answer
--------------------------------------------------------------------------------

<31>A 48-year-old man presents to his physician with complaints of dizziness
and fatigue. Physical examination reveals a blood pressure of 130/50 mm Hg
and a heart rate of 100 beats per minute. On examination, the physician
notes a large scar on the patient's abdomen. The man states that he was
severely injured in an automobile accident several years ago, and required
abdominal surgery at that time. Which of the following is the most likely
diagnosis?
A. Arteriovenous fistula
B. Cardiac tamponade
C. Heart failure
D. Hypovolemia
E. Shock
Answer
--------------------------------------------------------------------------------

<32>A renal pathologist examining the day's kidney biopsies notes that one
biopsy shows amorphous red nodules within the glomerular mesangium in
hematoxylin and eosin stained material. Congo red stain of the biopsy
demonstrates apple-green birefringence of these nodules. These nodules
are most likely to be related to which of the following?
A. Acute urinary tract infection
B. Diabetes mellitus
C. Sarcoidosis
D. Systemic lupus erythematosus
E. Tuberculosis
Answer
--------------------------------------------------------------------------------

<33>A patient goes to his family doctor complaining of persistent and
severe headaches. His physician diagnoses migraine headaches and prescribes
sumatriptan. What is the mechanism of action of this drug?
A. Dopamine1 agonist
B. GABAB antagonist
C. Muscarinic3 antagonist
D. Non-selective beta antagonist
E. Serotonin1D agonist
Answer
--------------------------------------------------------------------------------

<34>A 60-year-old man presents to his physician after a routine screening
test indicates hyperlipidemia. Physical examination reveals raised,
irregular, yellow papules in the skin of the soft tissues below the eyes.
Biopsy of these lesions would most likely show which of the following?
A. Benign nevus cells
B. Malignant nevus cells
C. Microscopic blisters
D. Munro microabscesses
E. Multinucleated giant cells
Answer
--------------------------------------------------------------------------------

<35> Trisomy 21 Normal karyotype
Positive test 100 50
Negative test 100 250
An experimental diagnostic test is developed to noninvasively detect the
presence of trisomy 21, Down's syndrome. The test is administered to a
group of 500 women considered to be at risk for a Down's fetus based on
blood tests. The results of this test are shown above. What is the
sensitivity of this new test?
A. 40%
B. 50%
C. 67%
D. 71%
E. 83%
Answer
--------------------------------------------------------------------------------

<36>An autopsy is performed on a 60-year-old man who developed progressive
dementia, parkinsonism, and visual hallucinations beginning 5 years prior
to death. Histopathologic examination reveals numerous eosinophilic
intracytoplasmic inclusions within neurons of substantia nigra, limbic
cortex, and basal nucleus of Meynert. These inclusions are immunoreactive
for ubiquitin. Which of the following is the most likely postmortem
diagnosis?
A. Alzheimer disease
B. Amyotrophic lateral sclerosis (ALS)
C. Diffuse Lewy body disease (dementia with Lewy bodies)
D. Parkinson disease
E. Pick disease
Answer
--------------------------------------------------------------------------------

<37>A 60-year-old factory worker reports to his physician that he can no
longer push heavy boxes across the floor. Upon examination it is noted
that the patient elevates the right shoulder when attempting shoulder
flexion, but shoulder abduction is not impaired. The medial border of
the right scapula is very prominent. Which of the following nerves
innervates the affected muscle?
A. Long thoracic nerve
B. Lower subscapular nerve
C. Musculocutaneous nerve
D. Suprascapular nerve
E. Upper subscapular nerve
Answer
--------------------------------------------------------------------------------

<38>The American Diabetes Association (ADA) recently lowered the cutoff
value for fasting glucose used in diagnosing diabetes mellitus from
140 mg/dL to 126 mg/dL. This reference interval change would be expected
to produce which of the following alterations?
A. Decrease the test's sensitivity
B. Increase the test's false negative rate
C. Increase the test's negative predictive value
D. Increase the test's positive predictive value
E. Increase the test's specificity
Answer
--------------------------------------------------------------------------------

<39>An emergency room physician examines a patient who has fallen from a
motorcycle and injured his shoulder. The clinician notices a loss of the
normal contour of the shoulder and a abnormal-appearing depression below
the acromion. Which of the following injuries did the patient most likely
sustain?
A. Avulsion of the coronoid process
B. Dislocated shoulder joint
C. Fracture of the mid shaft of the humerus
D. Fracture of the surgical neck of the humerus
E. Laceration of the axillary branch of the posterior cord
Answer
--------------------------------------------------------------------------------

<40>A 7-month-old child with failure to thrive is found to have a hemoglobin
of 4.4 g/dL. The peripheral smear shows very small red cells with marked
pallor. It is determined that the child has very low levels of
hemoglobin A, with elevated fractions of hemoglobin A2 and hemoglobin F.
Which of the following underlying mechanisms is most likely related to
the observed findings?
A. Amino acid substitution on b globin
B. Antibody against fetal blood cells
C. Cytoskeletal protein defect
D. Insufficient production of b globin
E. Iron deficiency
Answer
--------------------------------------------------------------------------------

<41>During a routine pelvic examination, a 20-year-old woman is found to
have an enlarged uterus. Ultrasound studies demonstrate a multiloculated
cystic structure within the uterine cavity, but no baby is identified.
This mass is removed with dilation and curettage of the uterus, and
placental-like tissue is observed during pathologic examination.
Which of the following tumor markers would be most useful in
establishing that residual tumor does not remain in the uterus?
A. b-hCG
B. Bombesin
C. CEA
D. PSA
E. S-100
Answer
--------------------------------------------------------------------------------

<42>On physical examination of a 7-year-old boy, the child's upper body
appears much more developed than his lower body. Blood pressure in the
upper extremities exceeds that of the lower extremities. On cardiac
examination, there is a midsystolic murmur over the anterior chest
and back. The child's lower extremities are cold, and femoral pulses
are absent. The part of the vascular system that is affected in this
disorder is derived from which of the following embryologic
structures?
A. Bulbus cordis
B. Ductus arteriosus
C. Left horn of sinus venosus
D. Right common cardinal vein
E. Right horn of sinus venosus
F. Third, fourth, and sixth aortic arches
Answer
--------------------------------------------------------------------------------

<43>A 42-year-old obese woman experiences episodic abdominal pain.
She notes that the pain increases after the ingestion of a fatty meal.
The action of which of the following hormones is responsible for the
postprandial intensification of her symptoms?
A. Cholecystokinin
B. Gastrin
C. Pepsin
D. Secretin
E. Somatostatin
Answer
--------------------------------------------------------------------------------

<44>A 42-year-old man was in an automobile accident and suffered severe
pelvic trauma. The bulb of the penis and the urethra were torn from the
inferior surface of the urogenital diaphragm. This has allowed urine to
extravasate from the urethra. Into which of the following regions will
the urine flow?
A. Deep perineal space
B. Ischioanal space
C. Rectovesical pouch
D. Retroperitoneal space
E. Superficial perineal space
Answer
--------------------------------------------------------------------------------

<45>Karyotypic analysis of a spontaneously aborted fetus demonstrates
trisomy of one of the chromosomes. Which chromosome is most likely to
be affected?
A. 8
B. 13
C. 16
D. 18
E. 21
Answer
--------------------------------------------------------------------------------

<46>At which of the following locations might a penetrating wound to the
heart damage the AV node?
A. Apex of the heart
B. Interatrial septum
C. Interventricular septum
D. Wall of the right atrium
E. Wall of the left atrium
Answer
--------------------------------------------------------------------------------

<47>A 9-month-old infant is brought to the Health Department to receive
the second dose of OPV (oral polio vaccine) 2 weeks after the first
vaccination. The child has mild diarrhea, so the decision is made to
defer further immunizations. Bacteriologic examination of a stool culture
is unremarkable; however, a small, single-stranded, positive RNA virus is
isolated from the specimen. This same agent was isolated from sewage
effluent the preceding week. The viral isolate was not inactivated by
ether. Which of the following viruses was most likely isolated?
A. Adenovirus
B. Hepatitis C
C. Parvovirus B19
D. Poliovirus
E. Rotavirus
Answer
--------------------------------------------------------------------------------

<48> The photomicrograph shows a section of synovium from the knee joint
of a patient with rheumatoid arthritis (RA). Which of the following are
the most abundant cells in the inflammatory infiltrate?
A. Eosinophils
B. Langhans type giant cells
C. Lymphocytes and plasma cells
D. Neutrophils
E. Type A and B synovial cells
Answer

--------------------------------------------------------------------------------
<49>A 54-year-old African-American female patient undergoes a routine
insurance physical examination. Chest x-ray reveals bilateral hilar
masses. Biopsy of the masses shows granulomata, but acid-fast and
fungal stains are negative for organisms. Which of the following
diseases should be suspected?
A. Caroli's disease
B. Raynaud's disease
C. Sarcoidosis
D. Scleroderma
E. Systemic lupus erythematosus
Answer
--------------------------------------------------------------------------------

<50>A neuroscientist discovers a way to selectively label neural crest
cells in a developing laboratory animal. After birth, he sacrifices the
animal and examines the tissue to search for labeled cells. Which of
the following cell types will contain the label?
A. Astrocytes
B. Ependymal cells
C. Microglia
D. Oligodendroglia
E. Pseudounipolar cells

Answer
--------------------------------------------------------------------------------

Answers
--------------------------------------------------------------------------------
1The correct answer is B. The most specific clue in the description is the presence of erythematous patches with central clearing, known clinically as "target lesions," which are associated with erythema multiforme. Both erythema multiforme and its severe, life-threatening version, known as Stevens-Johnson syndrome, are produced by immune complex deposition in dermal blood vessels. In approximately 50% of patients, no specific precipitating cause is identified. In the remainder of patients, however, a variety of causes have been implicated, including certain infections (herpes simplex, enteroviruses, Mycoplasma pneumoniae, Chlamydia, histoplasmosis), drugs (penicillin, sulfonamides, phenytoin, aspirin, corticosteroids, cimetidine, allopurinol, oral contraceptives), neoplasia (leukemia, lymphoma, multiple myeloma, internal malignancy), sarcoidosis, and foods (notably emulsifiers in margarine).
Erythema migrans chronicum (choice A) also produces an annular erythematous rash with central clearing, but usually affects the thigh, groin, and axilla; it is associated with Lyme disease.
Kaposi's sarcoma (choice C) causes purple lesions with no target lesions.
Psoriasis (choice D) causes erythematous plaques with silvery scale but does not produce target lesions.
Urticaria (choice E) causes wheals that are intensely pruritic, but does not produce target lesions.
--------------------------------------------------------------------------------2The correct answer is C. Raloxifene is a selective estrogen receptor modulator that helps prevent osteoporosis by lessening bone resorption and reducing bone turnover. It lowers risk for vertebral fractures by 40% to 50%. It is a bone-preserving alternative for women who prefer to avoid estrogen. Raloxifene does not cause breast pain and may lessen the risk for breast cancer in menopausal women. There is also a favorable effect on LDL and cholesterol.
Calcitonin nasal spray (choice A) is an osteoclastic bone resorption inhibitor that modestly increases bone mineral density and reduces the incidence of vertebral fracture. Although it is an estrogen alternative for bone preservation, its impact on hip fracture is not known. It is also lacks the anti-breast cancer properties of raloxifene.
Oral conjugated estrogen (choice B) and transdermal estradiol (choice E) are not the best choices, as this patient wants to avoid estrogen because of a strong family history of breast cancer. The route of administration of estrogen has been shown to have similar effects on bone preservation, even though the transdermal dosage is generally half that of the oral dosage. Breast cancer risk, however, is slightly increased with the unopposed higher dosage oral estrogen replacement.
Tamoxifen (choice D), while indicated in the long-term care of breast cancer patients, is not alone useful for treatment or prevention of osteoporosis. Tamoxifen is an anti-estrogen agent that competes with estrogen for binding sites.
--------------------------------------------------------------------------------3The correct answer is B. The nerve is the axillary branch of the posterior cord (also called the axillary nerve), which can be damaged by either fracture or dislocations of the humeral neck. The muscle is the deltoid muscle, which is supplied by this nerve. The deltoid arises from the lateral portions of the clavicle and the acromion and inserts into the deltoid tuberosity of the humerus.
The deltoid arises from both the acromion (choice A) and the clavicle.
The deltoid does not arise from the coracoid process (choices C, D, and E).
--------------------------------------------------------------------------------4The correct answer is B. The patient has neurosyphilis, specifically general paresis, a term that means "general paralysis of the insane." In this late sequela of syphilitic infection which occurs 5 to 20 years after infection, patients develop mental deterioration, which precedes motor system deterioration, leading eventually to "general paralysis" with mutism and incontinence. The abnormalities may be conveniently recalled using paresis as a mnemonic for personality, affect, hyperactive reflexes, Argyll Robertson pupils in the eyes, defects in the sensorium, intellectual decline and deficient speech. Specific anti-treponemal tests such as FTA-ABS are usually positive on both serum and cerebrospinal fluid.
The cerebrospinal (CSF) glucose (choice A) in neurosyphilis is usually normal.
Gram's stain (choice C) of CSF will not demonstrate spirochetes in neurosyphilis.
The CSF lymphocyte count (choice D) is typically elevated in neurosyphilis, but this is a non-specific finding.
The CSF neutrophil count (choice E) is usually normal in neurosyphilis.
--------------------------------------------------------------------------------
5>The correct answer is A. The man described is probably suffering from avoidant personality disorder, characterized by feelings of inadequacy and extreme sensitivity to criticism, leading to social inhibition and withdrawal. These individuals often avoid interpersonal relationships entirely rather than subject themselves to the potential risk of criticism or rejection, although they may yearn for a more satisfying personal life.
Borderline personality disorder (choice B) is characterized by unstable interpersonal relationships, instability of affect, impulsivity, feelings of emptiness or anger and, in some cases, paranoid or dissociative symptoms.
Dependent personality disorder (choice C) is characterized by the need for constant support and reassurance, with unrealistic anxieties over being forced to fend for oneself.
Narcissistic personality disorder (choice D) is characterized by excessive grandiosity and an exaggerated sense of self-importance, accompanied by a feeling of entitlement and a need for attention or admiration.
Schizotypal personality disorder (choice E) is characterized by eccentricities of behavior, odd beliefs or magical thinking, and difficulties with social and interpersonal relationships.

--------------------------------------------------------------------------------


--------------------------------------------------------------------------------6The correct answer is D. Respiratory syncytial virus is the most common cause of bronchiolitis and pneumonia in children younger than 1 year. Outbreaks occur seasonally in winter and early spring. Infection does not result in lasting immunity, and reinfection can occur.
Coronavirus (choice A) causes the common cold (nasal obstruction and discharge, sneezing, no fever or mild fever, occasional sore throat, and/or cough) and acute pharyngitis (sore throat, with or without cervical adenopathy, ulceration, and conjunctivitis).
Influenza type A (choice B) is the leading cause of influenza. Influenza is a systemic illness characterized by the sudden onset of fever, headache, myalgias, malaise, and prostration, followed by cough, nasal obstruction, and sore throat. The lower respiratory tract may also be involved.
Parainfluenza viruses (choice C) are the leading cause of croup, or acute laryngotracheobronchitis, in children. This infection involves both the upper and lower respiratory tracts. Inflammation in the subglottal area leads to hoarseness, dyspnea, a barking cough, and inspiratory stridor.
Rhinovirus (choice E) is the most common cause of the common cold.
--------------------------------------------------------------------------------7The correct answer is D. Maturation of germ cells (spermatogenesis) within the seminiferous tubules occurs in a concentric pattern with the less mature spermatogonia near the basal lamina and the mature forms near the tubule center. Spermatogonia are 2N cells and mature into larger primary spermatocytes (4N) (choice A). These mature into secondary spermatocytes (2N) (choice B), and finally into spermatids (1N) (choice C). Spermatids undergo spermiogenesis to become mature spermatozoa (choice E). Acrosomes form from the Golgi apparatus and a flagellum forms from microtubules. Unneeded organelles are shed. The seminiferous tubules of a reproductive-age male should exhibit all stages of maturation, with mature flagellated sperm in their centers.
--------------------------------------------------------------------------------
8The correct answer is C. The presentation described is classic for pulmonary hypertension, and, more specifically, the primary idiopathic form of pulmonary hypertension. This rare condition is suspected of being related to the collagen vascular diseases, since up to 50% of patients have antinuclear antibodies (despite the absence of frank presentation of other autoimmune disease). Also, a similar, known secondary form of pulmonary hypertension is sometimes seen in patients with a wide variety of collagen vascular diseases, including systemic lupus erythematosus, polymyositis, dermatomyositis, systemic sclerosis, and adult and juvenile forms of rheumatoid arthritis. A wide variety of other conditions have also been associated with secondary pulmonary hypertension, including shunts, left atrial hypertension, chronic hypoxia, pulmonary embolism, drug reaction, hepatic cirrhosis, and sickle cell disease. Both primary and secondary forms of pulmonary hypertension are associated with prominent changes in the pulmonary vasculature, which can include muscularization of smaller arterioles, concentric hypertrophy of the intima ("onion skinning"), and a distinctive plexiform lesion (plexogenic pulmonary vasculopathy) in which the smallest arterioles become markedly dilated with lumens partially occluded by endothelial (or possibly mesenchymal) cells and sometimes, thrombus. The prognosis of untreated pulmonary hypertension is poor. However, the use of the vasodilator hydralazine with anticoagulation can slow the course (fatal in about 3 years in untreated patients). If the pulmonary hypertension is secondary, therapy of the primary disease can be helpful.
Unlike cor pulmonale, atrial fibrillation with mural thrombus (choice A) formation is uncommon in primary pulmonary hypertension.
The absence of left ventricular findings on echocardiography tends to exclude myocardial infarction as the source of the patient's findings (choice B).
The presence of enlargement of the main pulmonary artery excludes pulmonary artery stenosis (choice D).
The clear lung fields exclude severe pulmonary fibrosis (choice E).
--------------------------------------------------------------------------------9The correct answer is B. The ureter leaves the renal pelvis and lies on the posterior abdominal wall as it descends to the pelvis. It crosses the pelvic brim at the level of the bifurcation of the common iliac artery. At this point it crosses anterior to the origin of the external iliac artery to enter the pelvis.
The origin of the common iliac artery (choice A) is at the bifurcation of the abdominal aorta, which occurs at the level of the fourth lumbar vertebra in the midline of the abdomen. Both ureters are lateral to the aortic bifurcation.
The origin of the internal iliac artery (choice C) is at the bifurcation of the common iliac artery, which occurs at about the level of the first sacral vertebra. The ureter and the internal iliac artery both enter the pelvis with the ureter on the lateral side of the artery.
The origin of the gonadal artery (choice D) (either the testicular artery or the ovarian artery) is from the abdominal aorta at a variable level, usually between the renal artery and the inferior mesenteric artery. The ureters are lateral to the origin of this artery.
The origin of the renal artery (choice E) is from the abdominal aorta at about the level of the second lumbar vertebra. The renal arteries pass laterally to enter the renal pelvis. At this site, the ureter is posterior to the artery.
--------------------------------------------------------------------------------10The correct answer is D. This is an example of relative polycythemia, in which there is an increased hematocrit or RBC count without a true increase in the total number of body RBCs. What usually happens in these cases is a significant reduction in plasma volume due to processes such as dehydration, vomiting, diarrhea, or diuresis.
Cyanotic heart disease (choice A), via appropriate erythropoietin secretion, can cause secondary absolute polycythemia.
Increased erythropoietin (choice B), whether appropriately or inappropriately secreted, can cause secondary absolute polycythemia.
Polycythemia vera (choice C) causes primary absolute polycythemia with usually low erythropoietin levels.
Renal cell carcinoma (choice E), via inappropriate erythropoietin secretion, can cause secondary absolute polycythemia.
--------------------------------------------------------------------------------11The correct answer is E. Hypospadias, which is congenital displacement of the urethral opening onto the ventral surface (underside) of the penis, is due to malformation of the urethral groove and canal. Hypospadias frequently accompanies other genitourinary anomalies, especially cryptorchidism. Isolated hypospadias is repaired because the abnormal opening is often constricted, leading to urinary retention and ascending urinary tract infections. Another important consequence of hypospadias is sterility, which occurs if the opening is too close to the base of the penis to permit normal ejaculation. Although hypospadias can occur in isolation, it is strongly associated with other urogenital anomalies.
Bladder exstrophy (choice A) is a completely unrelated congenital malformation in which the abdominal wall and anterior bladder wall form incompletely, and the bladder mucosa is exposed to the environment.
Hydrocele (choice B) is a serous accumulation in the tunica vaginalis, often producing a readily apparent scrotal mass. Hydrocele usually arises without any obvious cause.
Phimosis (choice C) is the condition in which the foreskin cannot be retracted over the head of the penis. It is usually either a congenital malformation or a consequence of scarring.
Urachal cysts (choice D) are produced by areas of persistent urachal epithelium, which may be present anywhere between the bladder and the umbilicus.
--------------------------------------------------------------------------------12The correct answer is E. Von Willebrand factor (vWF) is a self-polymerizing clotting protein present in the serum and the subendothelial basal lamina, which has binding sites for collagen, platelets, and fibrin. At a site of injury, vWF forms the bridge between the exposed collagen fibers and platelets in circulation, stimulating platelet degranulation and initiating the cellular component of the clotting cascade. An equally important role for vWF is binding platelets to the newly formed fibrin strands in a blood clot.
Factor VIII (choice A) and Factor IX (choice B) are clotting proteins of the intrinsic pathway. Factor VIII acts in concert with activated Factor IX (IXa) to cleave Factor X to Xa. Xa is the prothrombin activator central to both the intrinsic and extrinsic pathways.
Fibronectin (choice C) is a serum protein that acts as an opsonin for phagocytic cells in clots. Fibronectin binds non-specifically to bacteria and other materials in the newly forming clots, and draws the cell membrane of phagocytes into contact with these substances.
Tissue factor (choice D) is a protein released from injured tissues that works in concert with Factor VII to initiate the extrinsic pathway of coagulation. Like Factors VIII and IX, tissue factor and Factor VII cleave Factor X to Xa.
--------------------------------------------------------------------------------13The correct answer is D. The patient is most likely experiencing a potentiation of the effects of warfarin by norfloxacin, which decreases the metabolism of the warfarin. The increased warfarin effect produces an increase in the INR. (The target INR for patients with prosthetic heart valves is usually 1.5-4, depending on the type of valve.) Although norfloxacin is the most likely drug among the choices given to cause this effect in this patient, the antibiotics most commonly associated with this type of interaction are the macrolides, such as erythromycin, metronidazole, and the sulfonamide antibiotics.
Oral doses of penicillins, such as ampicillin (choice A), are generally not associated with a potentiation of warfarin's effect, although large IV doses of penicillin may be.
Cephalexin (choice B) is a first-generation cephalosporin that can be used in the treatment of acute cystitis. Although this agent is generally not associated with an increased hypoprothrombinemic effect when given with warfarin, the cephalosporins with a methyltetrazolethiol side chain, such as cefazolin, cefmetazole, and cefoperazone, are known to increase warfarin's therapeutic effect.
Nitrofurantoin (choice C) is a urinary anti-infective agent that does not interact with warfarin.
Phenazopyridine (choice E) is a urinary tract analgesic that does not interact with warfarin, although it commonly changes the color of urine to a bright orange/red color, which the patient may mistake as blood in the urine.
--------------------------------------------------------------------------------14The correct answer is D. Propylthiouracil works primarily by inhibiting the peripheral conversion of T4 to T3. The thyroid extracts iodide from the plasma and, in an oxidative process, iodinates tyrosine residues in thyroglobulin molecules. Monoiodotyrosines and diiodotyrosines are formed and then coupled to produce either thyroxine (tetraiodothyronine, T4) or triiodothyronine (T3). Proteolytic cleavage of thyroglobulin molecules leads to free T3 or T4, which is then released into the circulation; T3 is several times more potent than T4. Peripheral deiodination of T4 at the 5' position leads to T3 formation (mainly in the liver); this step is inhibited by propylthiouracil.
Decreasing the efficacy of TSH binding (choice A), decreasing the rate of thyroglobulin proteolysis (choice B), increasing the amount of rT3 formation (choice C), and inhibiting the uptake of iodide into the thyroid (choice E), would all tend to decrease the formation of thyroid hormones in the thyroid itself.
--------------------------------------------------------------------------------15The correct answer is A. Acetylcholine is the neurotransmitter of primary importance for the induction of REM sleep. Some of the other neurotransmitters do function in sleep, but REM sleep can occur in their absence.
Dopamine (choice B) is a neurotransmitter with a role in voluntary movement, mood, cognition, and regulation of prolactin release.
Epinephrine (choice C) is important in sympathetic nervous system responses. It is also a CNS neurotransmitter.
Norepinephrine (choice D) is important in sympathetic nervous system responses. It is also a CNS neurotransmitter involved in attention, arousal, and mood.
Serotonin (choice E) is a CNS neurotransmitter that plays an important role in mood and sensation. In the periphery, it is involved in vascular regulation and digestive function.
--------------------------------------------------------------------------------16The correct answer is E. This is a case of Neisseria gonorrhoeae arthritis. Patients are continuously susceptible to reinfection because of antigenic variation and phase variation of the pili.
N. gonorrhoeae is not an intracellular pathogen (choice A), although it may be found intracellularly in neutrophils after it has been phagocytized.
Ceftriaxone (choice B) is the drug of choice for N. gonorrhoeae.
Gonococci are especially susceptible to complement-mediated lysis, not resistant to it (choice C).
N. gonorrhoeae has an insignificant capsule that does not play a major role in the pathogenesis, but its capsule is immunogenic (compare with choice D). Streptococcus pyogenes is the best known example of a nonimmunogenic capsule, made of hyaluronic acid, but virtually all other capsules are immunogenic.
--------------------------------------------------------------------------------17The correct answer is E. Specificity is the proportion of persons without the disease who are correctly identified by the test as being disease-free. It is given by TN/(FP + TN), where TN stands for true negatives (people who do not have the disease and test negative on the test) and FP stands for false positives (people who do not have the disease but test positive on the test). In this case, 95/(5 + 95) = 95/100 = 0.95.
--------------------------------------------------------------------------------18The correct answer is B. 2,3-diphosphoglycerate (2,3-DPG) is produced in red cells by a variation on the glycolytic pathway, and levels diminish when glycolysis by the red cells slows. The depletion of 2,3-DPG in stored blood causes the hemoglobin dissociation curve to shift to the left, leading to an increase in oxygen affinity. This increase is helpful in the picking up of oxygen by hemoglobin in the lungs, but can be very problematic in the release of oxygen from the blood in tissues. This is not just a theoretical point: considerable effort has been expended in developing improved solutions for storing packed red cells and methods for "restoring" older stored cells so that the 2,3-DPG levels are adequate. In practice, in otherwise reasonably healthy patients, older transfused blood will quickly regenerate 2,3-DPG when placed in the glucose-containing environment of the serum, but even transiently decreased 2,3-DPG levels in a severely compromised patient can be dangerous.
--------------------------------------------------------------------------------19The correct answer is C. Entamoeba are relatively common enteric pathogens that can produce asymptomatic infection or more severe disease characterized by mucosal ulcerations and submucosal spread causing abdominal distress and liquid stools. Stools may show either trophozoite forms or the typical spherical cysts. Several species of Entamoeba are seen, including Entamoeba coli and E. hartmanni. E. histolytica cysts characteristically are spherical in shape, 10-20 mm in diameter, and have granular cytoplasm containing 1, 2, or 4 nuclei.
Cryptosporidium parvum (choice A) infections occur in the immunocompromised population and may cause severe diarrhea. The organism presents as minute (2-5 mm) intracellular spheres or arc-shaped merozoites under normal mucosa, and can be difficult to appreciate by light microscopy. Cysts in the stool are too small (4-5 mm) to be confused with Entamoeba.
Dientamoeba fragilis (choice B) is an intestinal amoeba that also may produce an infectious diarrhea. It does not have a cyst form, and only the trophozoite forms are seen in stools.
Giardia lamblia (choice D) is a flagellate protozoan that infects the stomach and small intestine when contaminated water is ingested. Diagnosis is usually made by examining duodenal contents; however, the stools may contain the oval or elliptical cysts, which are thick-walled and measure 8-14 mm in diameter. Spherical cysts are not seen in Giardia infection.
Isospora belli (choice E) produces self-limited intestinal infections mostly in the tropics, where fever and diarrhea may last weeks to months. The stool-borne cysts are much larger than Entamoeba (30 x 15 mm), are asymetrical, and are typically almond-shaped.
--------------------------------------------------------------------------------20The correct answer is A. The concrete operational stage (age 7-11 years) is defined by the child's awareness of the conservation of volume, which demonstrates that the child is able to reason in a logical way in terms of the physical world. Note that the child does not develop understanding of abstract concepts until he or she has reached the formal operational stage (choice B), at age 11 to adulthood.
The preoperational stage (choice C), ages 2 to 7 years, is associated with significant language development. However, the child has not yet developed the ability to take the perspective of others, and thus the child's thinking tends to remain egocentric.
The sensorimotor stage (choice D) corresponds to ages 0 to 2 years and is characterized by the infant developing increasingly sophisticated sensorimotor skills and behavior patterns.
--------------------------------------------------------------------------------21The correct answer is B. The reference intervals for serum alkaline phosphatase (ALP) and serum phosphate are higher in children than in adults. Growing bone in children requires alkaline phosphatase secreted by osteoblasts and an increase in serum phosphate to provide a proper solubility product ([calcium] x [phosphorus]) for mineralization of bone. However, the normal serum ALP in children is no greater than 5-times the upper limit of normal for adults, hence this patient's ALP is abnormally elevated. The most likely scenario is that the child is a victim of abuse and has suffered a femoral fracture that is healing. Bone repair further increases the serum ALP owing to the increase in osteoblastic activity.
Aseptic necrosis of bone in the leg (choice A) is more likely to occur in the femoral head (Legg-Calve-Perthes disease). This disease is more common in boys than girls and occurs in the 3- to 10-year-old age bracket. Although aseptic necrosis is associated with dense bone formation, it does not extend out into the soft tissue nor would it produce skin discoloration directly over the lesion (due to trauma in this case).
Since the elevation in serum ALP is increased for age and related to the bone lesion, choice C is incorrect.
Since the serum ALP is not normal for the age of this patient, choices D and E are incorrect. Furthermore, osteogenic sarcoma (choice D) does not normally occur in the first decade and favors the metaphysis of the distal femur or proximal tibia
--------------------------------------------------------------------------------22The correct answer is C. Norepinephrine (NE) is an agonist at alpha1-, alpha2- and beta1-receptors. NE exerts its vascular actions via alpha (predominantly alpha1) receptors and its cardiac actions via beta1-receptors. Labetalol is a nonselective antagonist at alpha- and beta-receptors, and therefore, could prevent all actions of NE.
Atenolol (choice A) is a selective beta1 antagonist, and therefore would block only norepinephrine's cardiac effects.
Esmolol (choice B) is a selective beta1 antagonist, and therefore would block only norepinephrine's cardiac effects. Metaproterenol (choice D) is a selective beta2 agonist and so would not block NE's effects.
Prazosin (choice E) is a selective alpha1 antagonist and would therefore block most of norepinephrine's actions in the vasculature, but would not antagonize other effects.
--------------------------------------------------------------------------------23The correct answer is C. The pudendal nerve is a branch of the sacral plexus that exits from the greater sciatic foramen and then enters the lesser sciatic foramen to lie in the pudendal canal on the lateral wall of the ischioanal fossa. The inferior rectal nerve is a branch of the pudendal nerve, which crosses the ischioanal fossa to reach the anal canal where it innervates the external anal sphincter and provides sensory innervation to the area.
The inferior gluteal nerve (choice A), a branch of the sacral plexus, exits through the greater sciatic foramen and innervates the gluteus maximus muscle.
The pelvic splanchnic nerves (choice B) are branches of the second, third, and fourth sacral nerves that carry preganglionic parasympathetic nerves to several pelvic organs.
The sciatic nerve (choice D) is a branch of the lumbosacral plexus that exits through the greater sciatic foramen, then enters the lower limb to innervate muscles of the posterior thigh, the leg, and the foot, and provides sensory innervation to the area as well.
The superior gluteal nerve (choice E) is a branch of the sacral plexus that exits through the greater sciatic foramen and innervates the gluteus medius, gluteus minimus, and tensor fascia latae muscles.
--------------------------------------------------------------------------------24The correct answer is A. The girl has Turner syndrome, which is specifically associated with coarctation of the aorta. Even when coarctation has not been diagnosed in early childhood and the patient appears to be doing well, surgical correction of any significant degree of coarctation is recommended. Uncorrected coarctation can cause death after about age 40 due to a variety of causes, including congestive heart failure, infective aortitis (analogous to infective endocarditis), and hypertension-induced intracranial hemorrhage or rupture of the damaged (pre-coarctation) aorta.
Associate dextrocardia (choice B) with Kartagener syndrome.
Associate ostium primum septal defect (choice C) with Down syndrome.
Pulmonary stenosis (choice D) and tetralogy of Fallot (choice E) are not specifically associated with Turner syndrome.
--------------------------------------------------------------------------------25The correct answer is A. The malignant tumor is a yolk sac tumor, also known as infantile embryonal carcinoma and endodermal sinus tumor. The biggest tip-off in the question stem is the presence of endodermal sinuses that resemble primitive glomeruli. The cytoplasmic globules described contain alpha-fetoprotein, indicating yolk cell differentiation, and alpha-1-antitrypsin. Alpha-fetoprotein can also be used as a serum marker for recurrent disease. Yolk sac tumors occur in pure form in infants and children and may be part of mixed tumors in adults.
Estrogen receptors (choice B) are important markers in breast cancer because they predict tumor response to hormonal manipulation.
Human chorionic gonadotropin (choice C) is found in the syncytial cells of embryonal carcinoma.
Human papilloma virus (choice D) can be found in condylomas, cervical cancer, penile cancer, laryngeal polyps, and warts.
Melanin (choice E) can be found in melanomas.
--------------------------------------------------------------------------------26The correct answer is D. The splenorenal ligament is the portion of the dorsal mesentery between the posterior abdominal wall and the spleen. This mesentery transmits the splenic artery and vein from their retroperitoneal position in the proximal portion of their course to the peritoneal spleen.
The gastrocolic ligament (choice A) is the portion of the greater omentum between the greater curvature of the stomach and the transverse colon. This portion of mesentery is not related to the spleen.
The gastrosplenic ligament (choice B) is the portion of the dorsal mesogastrium between the greater curvature of the stomach and the spleen. There are no splenic vessels in this mesentery.
The lesser omentum (choice C) is derived from the ventral mesentery. It is the mesentery between the lesser curvature of the stomach and the liver and between the first portion of the duodenum and the liver. It is not related to the spleen.
--------------------------------------------------------------------------------27The correct answer is D. This is one of those questions for which having a good idea of what you are looking for before exploring the answer choices will certainly save you valuable time. The answers all look alike and you could have been easily confused if you were not confident of the answer before approaching the choices.
This patient has long-standing hyperparathyroidism (elevated PTH), which predisposes to the development of osteitis fibrosa, her bone disease. PTH acts initially on osteocytes of bone tissue (osteocytic osteolysis) and subsequently on osteoclasts (osteoclastic resorption) to resorb calcium from bone matrix and make it available to the circulation. This increases plasma calcium levels. PTH also causes decreased phosphate reabsorption in the proximal renal tubule, yielding hypophosphatemia. Hypercalciuria is another sequela of excess PTH production, which predisposes the patient to the formation of calcium oxalate stones.
Choices A and E correspond to neither hyper- nor hypoparathyroid states.
Choice B is the profile of hypoparathyroidism. You should have quickly eliminated this choice since the PTH was decreased and you were looking for a profile consistent with HYPERparathyroidism.
Choice C is the profile of secondary hyperparathyroidism. This occurs when there is parathyroid overproduction due to a nonparathyroid cause. By far, the most common cause is chronic renal failure. In such cases, there is decreased calcium absorption since the kidneys are involved in the conversion of 25(OH)D3 to the active form 1,25(OH)D3. The decreased calcium ion level stimulates the parathyroid, leading to elevated PTH levels. Hyperphosphatemia results from diminished renal synthesis of 1,25 dihydroxyvitamin D3, creating further calcium-phosphate imbalance and enhanced PTH production.
--------------------------------------------------------------------------------28The correct answer is D. The processus vaginalis is an evagination of the parietal peritoneum of the abdomen that descends through the inguinal canal before the descent of the testis. Normally, the distal end of this evagination remains patent as the tunica vaginalis and the remainder of the processus vaginalis fuses and becomes fibrous. If a part of the processus vaginalis does not fuse (incomplete fusion), the resulting cystic structure is a hydrocele of the spermatic cord.
A communication between the epididymis and the tunica vaginalis (choice A) will result in the passage of sperm into the epididymis. This is a spermatocele.
Failure of the processus vaginalis to form (choice B) would result in an absence of the tunica vaginalis.
Total failure of the processus vaginalis to fuse (choice C) results in a persistent processus vaginalis. This results in congenital inguinal hernias.
Varicosities of the spermatic cord (choice E) or the pampiniform plexus is termed a varicocele.